Buy BOOKS at Discounted Price

Arithmetic Progressions

Class 11th Mathematics RD Sharma Solution
Exercise 19.1
  1. If the nth term of a sequence is given by an = n^2 - n+1, write down its first…
  2. A sequence is defined by an = n^3 - 6n^2 + 11n - 6, n ∈ N. Show that the first…
  3. Find the first four terms of the sequence defined by a1 = 3 and an = 3an-1 + 2,…
  4. a1 = 1, an = an-1 + 2, n 1 Write the first five terms in each of the following…
  5. a1 = 1 = a2, an = an-1 + an-2, n2 Write the first five terms in each of the…
  6. a1 = a2 =2, an = an-1 - 1, n2 Write the first five terms in each of the…
  7. The Fibonacci sequence is defined by a1 = 1 = a2, an = an-1 + an-2for n 2. Find…
  8. 3, -1, -5, -9… Show that each of the following sequences is an A.P. Also, find…
  9. -1 , 1/4 , 3/2 , 11/4 , l Show that each of the following sequences is an A.P.…
  10. root 2 , 3 root 2 , 5 root 2 , 7 root 2 , l l Show that each of the following…
  11. 9, 7, 5, 3 … Show that each of the following sequences is an A.P. Also, find…
  12. The nth term of a sequence is given by an = 2n + 7. Show that it is an A.P.…
  13. The nth term of a sequence is given by an = 2n^2 + n+ 1. Show that it is not an…
Exercise 19.2
  1. 10th term of the A.P. 1, 4, 7,10,..... Find:
  2. 18th term of the A.P. root 2 , 3 root 2 , 5 root 2 Find:
  3. nth term of the A.P. 13, 8, 3, -2,…… Find:
  4. In an A.P., show that am+n + am-n = 2am.
  5. Which term of the A.P. 3, 8, 13,… is 248 ?
  6. Which term of the A.P. 84, 80, 76,… is 0 ?
  7. Which term of the A.P. 4, 9, 14,… is 254 ?
  8. Is 68 a term of the A.P. 7, 10, 13,…?
  9. Is 302 a term of the A.P. 3, 8, 13,…?
  10. Which term of the sequence 24 , 23 1/4 , 22 1/2 , 21 3/4 is the first negative…
  11. Which term of the sequence 12 + 8i, 11 + 6i, 10 + 4i, … is (a) purely real (b)…
  12. How many terms are in A.P. 7, 10, 13,…43?
  13. How many terms are there in the A.P. -1 , - 5/6 , - 2/3 , - 1/2 , l , 10/3 ?…
  14. The first term of an A.P. is 5, the common difference is 3, and the last term…
  15. The 6th and 17th terms of an A.P. are 19 and 41 respectively. Find the 40th…
  16. If 9th term of an A.P. is Zero, prove that its 29th term is double the 19th…
  17. If 10 times the 10th term of an A.P. is equal to 15 times the 15th term, show…
  18. The 10th and 18th term of an A.P. are 41 and 73 respectively, find 26th term.…
  19. In a certain A.P. the 24th term is twice the 10th term. Prove that the 72nd…
  20. If (m + 1)th term of an A.P. is twice the (n + 1)th term, prove that (3m +…
  21. If the nth term of the A.P. 9, 7, 5,… is same as the nth term of the A.P. 15,…
  22. 3, 5, 7, 9, … 201 Find the 12th term from the end of the following arithmetic…
  23. 3, 8, 13, … 253 Find the 12th term from the end of the following arithmetic…
  24. 1, 4, 7, 10, … 88 Find the 12th term from the end of the following arithmetic…
  25. The 4th term of an A.P. is three times the first and the 7th term exceeds…
  26. Find the second term and nth term of an A.P. whose 6th term is 12 and 8th term…
  27. How many numbers of two digit are divisible by 3?
  28. An A.P. consists of 60 terms. If the first and the last terms be 7 and 125…
  29. The sum of the 4th and the 8th terms of an A.P. is 24, and the sum of the 6th…
  30. How many numbers are there between 1 and 1000 which when divided by 7 leave…
  31. The first and the last term of an A.P. are a and l respectively. Show that the…
  32. If an A.P. is such that a_4/a_7 = 2/3 find a_ delta/a_8 .
  33. If θ1, θ2, θ3, …, θn are in AP, whose common difference is d, show that…
Exercise 19.3
  1. The Sum of the three terms of an A.P. is 21 and the product of the first, and…
  2. Three numbers are in A.P. If the sum of these numbers be 27 and the product…
  3. Find the four numbers in A.P., whose sum is 50 and in which the greatest number…
  4. The sum of three numbers in A.P. is 12, and the sum of their cubes is 288. Find…
  5. If the sum of three numbers in A.P. is 24 and their product is 440, find the…
  6. The angles of a quadrilateral are in A.P. whose common difference is 10. Find…
Exercise 19.4
  1. 50, 46, 42, …. To 10 terms Find the sum of the following arithmetic…
  2. 1, 3, 5, 7, … to 12 terms Find the sum of the following arithmetic…
  3. 3, 9/2 , 6, 15/2 , … to 25 terms Find the sum of the following arithmetic…
  4. 41, 36, 31, … to 12 terms Find the sum of the following arithmetic…
  5. a + b, a - b, a - 3b, … to 22 terms Find the sum of the following arithmetic…
  6. Find the sum of the following arithmetic progressions: (x-y)^2 , (x^2 +y^2),…
  7. x-y/x+y , 3x-2y/x+y , 5x-3y/x+y , … to n terms Find the sum of the following…
  8. 2 + 5 + 8 + … + 182 Find the sum of the following series:
  9. 101 + 99 + 97 + … + 47 Find the sum of the following series:
  10. (a - b)^2 + (a^2 + b^2) + (a + b)^2 + s…. + [(a + b)^2 + 6ab] Find the sum of…
  11. Find the sum of first n natural numbers.
  12. Find the sum of all - natural numbers between 1 and 100, which are divisible by…
  13. Find the sum of first n odd natural numbers.
  14. Find the sum of all odd numbers between 100 and 200
  15. Show that the sum of all odd integers between 1 and 1000 which are divisible by…
  16. Find the sum of all integers between 84 and 719, which are multiples of 5…
  17. Find the sum of all integers between 50 and 500 which are divisible by 7…
  18. Find the sum of all even integers between 101 and 999
  19. Find the sum of all integers between 100 and 550, which are divisible by 9…
  20. Find the sum of the series: 3 + 5 + 7 + 6 + 9 + 12 + 9 + 13 + 17 + … to 3n…
  21. Find the sum of all those integers between 100 and 800 each of which on…
  22. 25 + 22 + 19 + 16 + … + x = 115 Solve:
  23. 1 + 4 + 7 + 10 + …. + x = 590 Solve:
  24. Find the rthterm of an A.P., the sum of whose first n terms is 3n^2 +2n…
  25. How many terms are there in the A.P. whose first and fifth terms are - 14 and…
  26. The sum of the first 7 terms of an A.P. is 10, and that of the next 7 terms is…
  27. The third term of an A.P. is 7, and the seventh term exceeds 3 times the third…
  28. The first term of an A.P. is 2, and the last term is 50. The sum of all these…
  29. The number of terms of an A.P. is even; the sum of the odd term is 24, of the…
  30. If Sn = n^2 p and s_m = m^2p , m not equal n in an A.P., prove that Sp = p^3…
  31. If the 12th term of an A.P. is - 13 and the sum of the first 4 terms is 24,…
  32. If the 5th and 12th terms of an A.P. are 30 and 65 respectively, what is the…
  33. Find the sum of n terms of the A.P. whose kth terms is 5k + 1
  34. Find the sum of all two digit numbers which when divided by 4, yields 1 as the…
  35. If the sum of a certain number of terms of the A.P. 25, 22, 19, …. is 116.…
  36. Find the sum of odd integers from 1 to 2001.
  37. How many terms of the A.P. - 6, - 11/2 , - 5, …, are needed to give the sum -…
  38. In an A.P. the first term is 2, and the sum of the first 5 terms is one-fourth…
  39. If S1 be the sum of (2n + 1) terms of an A.P. and S2 sum of its odd terms,…
  40. Find an A.P. in which the sum of any number of terms is always three times the…
  41. If the sum of n terms of an A.P. is nP + 1/2 n(n - 1)Q where P and Q are…
  42. The sums of n terms of two arithmetic progressions are in the ratio 5n + 4: 9n…
  43. The sum of first n terms of two A.P.’s is in the ratio (7n + 2) : (n + 4).…
Exercise 19.5
  1. If 1/a , 1/b , 1/c are in A.P., prove that: b+c/a , c+a/b , a+b/c are in A.P.…
  2. If 1/a , 1/b , 1/c are in A.P., prove that: a(b + c), b(c + a), c(a + b) are…
  3. If a^2 , b^2 , c^2 are in A.P., prove that a/b+c , b/c+a , c/a+b are in A.P.…
  4. If a, b, c are in A.P., then show that: (i) a^2 (b + c), b^2 (c + a), c^2 (a +…
  5. If b+c/a , c+a/b , a+b/c are in A.P., prove that: 1/a , 1/b , 1/c are in A.P.…
  6. If b+c/a , c+a/b , a+b/c are in A.P., prove that: bc, ca, ab are in A.P.…
  7. (a - c)^2 = 4 (a - b) (b - c) If a, b, c are in A.P., prove that:…
  8. a^2 + c^2 + 4ac = 2 (ab + bc + ca) If a, b, c are in A.P., prove that:…
  9. a^3 + c^3 + 6abc = 8b^3 If a, b, c are in A.P., prove that:
  10. If a (1/b + 1/c) , b (1/c + 1/a) , c (1/a + 1/b) are in A.P., prove that a, b,…
  11. Show that x^2 + xy + y^2 , z^2 + zx + x^2 and y^2 + yz + z^2 are in consecutive…
Exercise 19.6
  1. Find the A.M. between: (i) 7 and 13 (ii) 12 and - 8 (iii) (x - y) and (x + y)…
  2. Insert 4 A.M.s between 4 and 19.
  3. Insert 7 A.M.s between 2 and 17.
  4. Insert six A.M.s between 15 and - 13.
  5. There are n A.M.s between 3 and 17. The ratio of the last mean to the first…
  6. Insert A.M.s between 7 and 71 in such a way that the 5th A.M. is 27. Find the…
  7. If n A.M.s are inserted between two numbers, prove that the sum of the means…
  8. If x, y, z are in A.P. and A1is the A.M. of x and y, and A2 is the A.M. of y…
  9. Insert five numbers between 8 and 26 such that the resulting sequence is an A.P…
Exercise 19.7
  1. A man saved ₹ 16500/- in ten years . in each year after the first he saved ₹…
  2. A man saves ₹ 32 during the first year, ₹ 36 in the second year and in this way…
  3. A man arranges to pay off a debt of ₹ 3600 by 40 annual instalments which form…
  4. A manufacturer of the radio sets produced 600 units in the third year and 700…
  5. There are 25 trees at equal distances of 5 meters in a line with a well, the…
  6. A man is employed to count ₹ 10710. He counts at the rate of ₹ 180 per minute…
  7. A piece of equipment cost a certain factory ₹ 600, 000. If it depreciates in…
  8. A farmer buys a used tractor for ₹ 12000. He pays ₹ 6000 cash and agrees to pay…
  9. Shamshad Ali buys a Scooter for ₹ 22000. He pays ₹ 4000 cash and agrees to pay…
  10. The income of a person is ₹ 300, 000 in the first year and he receives an…
  11. A man starts repaying a loan as first instalment of Rs.100. If he increases…
  12. A carpenter was hired to build 192 window frames. The first day he made five…
  13. We know that the sum of interior angles of a triangle is 180°. Show that the…
  14. In a potato race 20 potato are placed in a line at intervals of 4 meters with…
  15. A man accepts a position with an initial salary of ₹ 5200 per month. It is…
  16. A man saved ₹ 66000 in 20 years. In each succeeding year after the first year…
  17. In a cricket team tournment 16 teams particpated a sum of 8000 is to to…
Very Short Answer
  1. Write the common difference of an A.P. whose nth term is xn + y.
  2. Write the common difference of an A.P. the sum of whose first n terms is {p}/{2}…
  3. If the sum of n terms of an AP is 2n^{2} + 3n , then write its nth term.…
  4. If log 2, log ( 2^{x} - 1 ) and log ( 2^{x} + 3 ) are in A.P., write the value of…
  5. If the sum of n terms of two arithmetic progressions are in the ratio 2n + 5 : 3n + 4,…
  6. Write the sum of first n odd natural numbers.
  7. Write the sum of first n even natural numbers.
  8. Write the value of n for which nth of the A.P.s 3, 10, 17, ... and 63, 65, 67, ... are…
  9. If { 3+5+7 + l. s + n }/{ 5+8+11 + l 10 } = 7 , then find the value of n.…
  10. If mth term of an A.P. is n and nth term is m, then write its pth term.…
  11. If the sum of n terms of two A.P.’s are in the ratio (3n + 2) : (2n + 3), find the…
Mcq
  1. If 7th and 13th terms of an A.P. be 34 and 64 respectively, then its 18th term is Mark…
  2. If the sum of p terms of an A.P. is q and the sum of q terms is p, then the sum or p +…
  3. If the sum of n terms of an A.P. be 3 n^{2} - n and its common difference is 6, then…
  4. Sum of all two digit numbers which when divided by 4 yield unity as remainder is Mark…
  5. In A.M.’s are introduced between 3 and 17 such that the ratio of the last mean to the…
  6. If s_{n} denotes the sum of first n terms of an A.P. such that {s_{m}}/{s_{n}}…
  7. The first and last terms of an A.P. are 1 and 11. If the sum of its terms is 36, then…
  8. If the sum of n terms of an A.P., is 3 n^{2} + 5n which of its terms is 164? Mark the…
  9. If the sum of n terms of an A.P. is 2 n^{2} + 5n , then its nth term is Mark the…
  10. If a_{1} , a_{2} , a_{3} , . sa_{n} are in A.P. with common difference d, then the…
  11. In the arithmetic progression whose common difference is non-zero, the sum of first 3…
  12. If a_{1} , a_{2} , a_{3} , . sa_{n} are in A.P. with common difference d, then the…
  13. If four numbers in A.P. are such that their sum is 50 and the greatest number is 4…
  14. In n arithmetic means are inserted between 1 and 31 such that the ratio of the first…
  15. Let s_{th} denote the sum of n terms of an A.P. whose first term is a. If the common…
  16. The first and last term of an A.P. are a and l respectively. If S is the sum of all…
  17. If the sum of first n even natural numbers is equal to k times the sum of first n odd…
  18. If the first, second and last term of an A.P. are a, b and 2a respectively, then its…
  19. If, s_{1} is the sum of an arithmetic progression of ‘n’ odd number of terms and…
  20. If in an A.P., s_{n} = n^{2}p and s_{m} = m^{2}p , where s_{f} denotes the sum of…
  21. If in an A.P., the pth term is q and (p+q)^ term is zero, then the q^ term is…
  22. The 10th common term between the A.P.s 3, 7, 11, 15, ... and 1, 6, 11, 16, ... is Mark…
  23. If in an A.P. s_{n} = n^{2} and s_{m} = m^{2} , where s_{f} denotes the sum r…
  24. Let s_{th} denote the sum of first n terms of an A.P. If s_{2n} = 3s_{n} , then…

Exercise 19.1
Question 1.

If the nth term of a sequence is given by an = n2 – n+1, write down its first five terms.


Answer:

Given,


an = n2 – n+1


We can find first five terms of this sequence by putting values of n from 1 to 5.


When n = 1:


a1 = (1)2 – 1 + 1


⇒ a1 = 1 – 1 + 1


⇒ a1 = 1


When n = 2:


a2 = (2)2 – 2 + 1


⇒ a2 = 4 – 2 + 1


⇒ a2 = 3


When n = 3:


a3 = (3)2 – 3 + 1


⇒ a3 = 9 – 3 + 1


⇒ a3 = 7


When n = 4:


a4 = (4)2 – 4 + 1


⇒ a4 = 16 – 4 + 1


⇒ a4 = 13


When n = 5:


a5 = (5)2 – 5 + 1


⇒ a5 = 25 – 5 + 1


⇒ a5 = 21


∴ First five terms of the sequence are 1, 3, 7, 13, 21.



Question 2.

A sequence is defined by an = n3 – 6n2 + 11n – 6, n ∈ N. Show that the first three terms of the sequence are zero and all other terms are positive.


Answer:

Given,


an = n3 – 6n2 + 11n – 6, n ∈ N


We can find first three terms of sequence by putting the values of n form 1 to 3.


When n = 1:


a1 = (1)3 – 6(1)2 + 11(1) – 6


⇒ a1 = 1 – 6 + 11 – 6


⇒ a1 = 12 – 12


⇒ a1 = 0


When n = 2:


a2 = (2)3 – 6(2)2 + 11(2) – 6


⇒ a2 = 8 – 6(4) + 22 – 6


⇒ a2 = 8 – 24 + 22 – 6


⇒ a2 = 30 – 30


⇒ a2 = 0


When n = 3:


a3 = (3)3 – 6(3)2 + 11(3) – 6


⇒ a3 = 27 – 6(9) + 33 – 6


⇒ a3 = 27 – 54 + 33 – 6


⇒ a3 = 60 – 60


⇒ a3 = 0


This shows that the first three terms of the sequence is zero.


When n = n:


an = n3 – 6n2 + 11n – 6


⇒ an = n3 – 6n2 + 11n – 6 – n + n – 2 + 2


⇒ an = n3 – 6n2 + 12n – 8 – n + 2


⇒ an = (n)3 – 3×2n(n – 2) – (2)3 – n + 2


{(a – b)3 = (a)3 – (b)3 – 3ab(a – b)}


⇒ an = (n – 2)3 – (n – 2)


Here, n – 2 will always be positive for n > 3


∴ an is always positive for n > 3



Question 3.

Find the first four terms of the sequence defined by a1 = 3 and an = 3an–1 + 2, for all n > 1.


Answer:

Given,


a1 = 3 and an = 3an–1 + 2, for all n > 1


We can find the first four terms of a sequence by putting values of n


from 1 to 4


When n = 1:


a1 = 3


When n = 2:


a2 = 3a2–1 + 2


⇒ a2 = 3a1 + 2


⇒ a2 = 3(3) + 2


⇒ a2 = 9 + 2


⇒ a2 = 11


When n = 3:


a3 = 3a3–1 + 2


⇒ a3 = 3a2 + 2


⇒ a3 = 3(11) + 2


⇒ a3 = 33 + 2


⇒ a3 = 35


When n = 4:


a4 = 3a4–1 + 2


⇒ a4 = 3a3 + 2


⇒ a4 = 3(35) + 2


⇒ a4 = 105 + 2


⇒ a4 = 107


∴ First four terms of sequence are 3, 11, 35, 107.



Question 4.

Write the first five terms in each of the following sequences:

a1 = 1, an = an–1 + 2, n > 1


Answer:

Given,


a1 = 1, an = an–1 + 2, n > 1


We can find the first five terms of a sequence by putting values of n


from 1 to 5


When n = 1:


a1 = 1


When n = 2:


a2 = a2–1 + 2


⇒ a2 = a1 + 2


⇒ a2 = 1 + 2


⇒ a2 = 3


When n = 3:


a3 = a3–1 + 2


⇒ a3 = a2 + 2


⇒ a3 = 3 + 2


⇒ a3 = 5


When n = 4:


a4 = a4–1 + 2


⇒ a4 = a3 + 2


⇒ a4 = 5 + 2


⇒ a4 = 7


When n = 5:


a5 = a5–1 + 2


⇒ a5 = a4 + 2


⇒ a5 = 7 + 2


⇒ a5 = 9


∴ First five terms of the sequence are 1, 3, 5, 7, 9.



Question 5.

Write the first five terms in each of the following sequences:

a1 = 1 = a2, an = an–1 + an–2, n>2


Answer:

Given,


a1 = 1 = a2, an = an–1 + an–2, n>2


We can find the first five terms of a sequence by putting values of n


from 1 to 5


When n = 1:


a1 = 1


When n = 2:


a1 = 1


When n = 3:


a3 = a3–1 + a3–2


⇒ a3 = a2 + a1


⇒ a3 = 1 + 1


⇒ a3 = 2


When n = 4:


a4 = a4–1 + a4–2


⇒ a4 = a3 + a2


⇒ a4 = 2 + 1


⇒ a4 = 3


When n = 5:


a5 = a5–1 + a5–2


⇒ a5 = a4 + a3


⇒ a5 = 3 + 2


⇒ a5 = 5


∴ First five terms of the sequence are 1, 1, 2, 3, 5.



Question 6.

Write the first five terms in each of the following sequences:

a1 = a2 =2, an = an–1 – 1, n>2


Answer:

Given,


a1 = 2 = a2, an = an–1 – 1, n>2


We can find the first five terms of a sequence by putting values of n


from 1 to 5


When n = 1:


a1 = 2


When n = 2:


a1 = 2


When n = 3:


a3 = a3–1 – 1


⇒ a3 = a2 – 1


⇒ a3 = 2 – 1


⇒ a3 = 1


When n = 4:


a4 = a4–1 – 1


⇒ a4 = a3 – 1


⇒ a4 = 1 – 1


⇒ a4 = 0


When n = 5:


a5 = a5–1 – 1


⇒ a5 = a4 – 1


⇒ a5 = 0 – 1


⇒ a5 = -1


∴ First five terms of the sequence are 2, 2, 1, 0, -1.



Question 7.

The Fibonacci sequence is defined by a1 = 1 = a2, an = an–1 + an–2for n > 2. Find for n = 1, 2, 3, 4, 5.


Answer:

Given: a1 = 1 = a2, an = an–1 + an–2, n>2


When n = 1:



a3 = a3–1 + a3–2


⇒ a3 = a2 + a1


⇒ a3 = 1 + 1


⇒ a3 = 2


When n = 2:



a4 = a4–1 + a4–2


⇒ a4 = a3 + a2


⇒ a4 = 2 + 1


⇒ a4 = 3


When n = 3:



a5 = a5–1 + a5–2


⇒ a5 = a4 + a3


⇒ a5 = 3 + 2


⇒ a5 = 5


When n = 4:



a6 = a6–1 + a6–2


⇒ a6 = a5 + a4


⇒ a6 = 5 + 3


⇒ a6 = 8


When n = 5:





Question 8.

Show that each of the following sequences is an A.P. Also, find the common difference and write 3 more terms in each case.

3, -1, -5, -9…


Answer:

A.P is known for Arithmetic Progression whose common difference = an – an-1 where n > 0


a1 = 3, a2 = -1, a3 = -5, a4 = -9


Now, a2 – a1 = -1 – 3 = -4


a3 – a2 = -5 – (-1) = -5 + 1 = -4


a4 – a3 = -9 – (-5) = -9 + 5 = -4


As, a2 – a1 = a3 – a2 = a4 – a3


The given sequence is A.P


Common difference, d = a2 – a1 = -4


To find next three more terms of A.P, firstly find an


We know, an = a + (n-1) d where a is first term or a1 and d is common difference


∴ an = 3 + (n-1) -4


⇒ an = 3 – 4n + 4


⇒ an = 7 – 4n


When n = 5:


a5 = 7 – 4(5)


⇒ a5 = 7 – 20


⇒ a5 = -13


When n = 6:


a6 = 7 – 4(6)


⇒ a6 = 7 – 24


⇒ a6 = -17


When n = 7:


a7 = 7 – 4(7)


⇒ a7 = 7 – 28


⇒ a7 = -21


Hence, A.P is 3, -1, -5, -9, -13, -17, -21,…



Question 9.

Show that each of the following sequences is an A.P. Also, find the common difference and write 3 more terms in each case.



Answer:

A.P is known for Arithmetic Progression whose common difference = an – an-1 where n > 0






As, a2 – a1 = a3 – a2 = a4 – a3


The given sequence is A.P


Common difference, d


To find next three more terms of A.P, firstly find an


We know, an = a + (n-1) d where a is first term or a1 and d is common difference







When n = 5:





⇒ a5 = 4


When n = 6:





When n = 7:








Question 10.

Show that each of the following sequences is an A.P. Also, find the common difference and write 3 more terms in each case.



Answer:

A.P is known for Arithmetic Progression whose common difference = an – an-1 where n > 0






As, a2 – a1 = a3 – a2 = a4 – a3


The given sequence is A.P


Common difference,d


To find the next three more terms of A.P, firstly find an


We know, an = a + (n-1) d where a is a first term or a1 and d is common difference





When n = 5:





When n = 6:





When n = 7:





Hence, A.P. is



Question 11.

Show that each of the following sequences is an A.P. Also, find the common difference and write 3 more terms in each case.

9, 7, 5, 3 …


Answer:

A.P is known for Arithmetic Progression whose common difference = an – an-1 where n > 0


a1 = 9, a2 = 7, a3 = 5, a4 = 3


Now, a2 – a1 = 7 – 9 = -2


a3 – a2 = 5 – 7 = -2


a4 – a3 = 3 – 5 = -2


As, a2 – a1 = a3 – a2 = a4 – a3


The given sequence is A.P


Common difference, d = a2 – a1 = -2


To find the next three more terms of A.P, firstly find an


We know, an = a + (n-1) d where a is first term or a1 and d is common difference


∴ an = 9 + (n-1) -2


⇒ an = 9 – 2n + 2


⇒ an = 11 – 2n


When n = 5:


a5 = 11 – 2(5)


⇒ a5 = 11 – 10


⇒ a5 = 1


When n = 6:


a6 = 11 – 2(6)


⇒ a6 = 11 – 12


⇒ a6 = -1


When n = 7:


a7 = 11 – 2(7)


⇒ a7 = 11 – 14


⇒ a7 = -3


Hence, A.P is 9, 7, 5, 3, 1, -1, -3,….



Question 12.

The nth term of a sequence is given by an = 2n + 7. Show that it is an A.P. Also, find its 7th term.


Answer:

Given,


an = 2n + 7


We can find first five terms of this sequence by putting values of n from 1 to 5.


When n = 1:


a1 = 2(1) + 7


⇒ a1 = 2 + 7


⇒ a1 = 9


When n = 2:


a2 = 2(2) + 7


⇒ a2 = 4 + 7


⇒ a2 = 11


When n = 3:


a3 = 2(3) + 7


⇒ a3 = 6 + 7


⇒ a3 = 13


When n = 4:


a4 = 2(4) + 7


⇒ a4 = 8 + 7


⇒ a4 = 15


When n = 5:


a5 = 2(5) + 7


⇒ a5 = 10 + 7


⇒ a5 = 17


∴ First five terms of the sequence are 9, 11, 13, 15, 17.


A.P is known for Arithmetic Progression whose common difference = an – an-1 where n > 0


a1 = 9, a2 = 11, a3 = 13, a4 = 15, a5 = 17


Now, a2 – a1 = 11 – 9 = 2


a3 – a2 = 13 – 11 = 2


a4 – a3 = 15 – 13 = 2


a5 – a4 = 17 – 15 = 2


As, a2 – a1 = a3 – a2 = a4 – a3 = a5 – a4


The given sequence is A.P


Common difference, d = a2 – a1 = 2


To find the seventh term of A.P, firstly find an


We know, an = a + (n-1) d where a is first term or a1 and d is common difference


∴ an = 3 + (n-1) 2


⇒ an = 3 + 2n – 2


⇒ an = 2n + 1


When n = 7:


a7 = 2(7) + 1


⇒ a7 = 14 + 1


⇒ a7 = 15


Hence, the 7th term of A.P. is 15



Question 13.

The nth term of a sequence is given by an = 2n2 + n+ 1. Show that it is not an A.P.


Answer:

Given,


an = 2n2 + n + 1


We can find first three terms of this sequence by putting values of n from 1 to 3.


When n = 1:


a1 = 2(1)2 + 1 + 1


⇒ a1 = 2(1) + 2


⇒ a1 = 2 + 2


⇒ a1 = 4


When n = 2:


a2 = 2(2)2 + 2 + 1


⇒ a2 = 2(4) + 3


⇒ a2 = 8 + 3


⇒ a2 = 11


When n = 3:


a3 = 2(3)2 + 3 + 1


⇒ a3 = 2(9) + 4


⇒ a3 = 18 + 4


⇒ a3 = 22


∴ First three terms of the sequence are 4, 11, 22.


A.P is known for Arithmetic Progression whose common difference = an – an-1 where n > 0


a1 = 4, a2 = 11, a3 = 22


Now, a2 – a1 = 11 – 4 = 7


a3 – a2 = 22 – 11 = 11


As a2 – a1 is not equal to a3 – a2


The given sequence is not an A.P.




Exercise 19.2
Question 1.

Find:

10th term of the A.P. 1, 4, 7,10,.....


Answer:

A.P is known for Arithmetic Progression whose common difference = an – an-1 where n > 0


a = a1 = 1, a2 = 4


Common difference, d = a2 – a1 = 4 – 1 = 3


To find the tenth term of A.P, firstly find an


We know, an = a + (n-1) d where a is first term or a1 and d is common difference


∴ an = 1 + (n-1) 3


⇒ an = 1 + 3n – 3


⇒ an = 3n – 2


When n = 10:


a10 = 3(10) – 2


⇒ a10 = 30 – 2


⇒ a10 = 28


Hence, 10th term of A.P. is 28



Question 2.

Find:

18th term of the A.P.


Answer:

A.P is known for Arithmetic Progression whose common difference = an – an-1 where n > 0



Common difference, d


To find 18th term of A.P, firstly find an


We know, an = a + (n-1) d where a is a first term or a1 and d is common difference





When n = 18:





Hence, 18th term of A.P. is 34√2



Question 3.

Find:

nth term of the A.P. 13, 8, 3, -2,……


Answer:

A.P is known for Arithmetic Progression whose common difference = an – an-1 where n > 0


a = a1 = 13, a2 = 8


Common difference, d = a2 – a1 = 8 – 13 = -5


We know, an = a + (n-1) d where a is first term or a1 and d is common difference


∴ an = 13 + (n-1) -5


⇒ an = 13 – 5n + 5


⇒ an = 18 – 5n


Hence, nth term of A.P. is 18 – 5n



Question 4.

In an A.P., show that am+n + am–n = 2am.


Answer:

Let common difference of an A.P is d and first term is a


We know, an = a + (n – 1)d where a is first term or a1 and d is common difference


Now, take L.H.S.:


am+n + am-n = a + (m + n – 1)d + a + (m - n – 1)d


⇒ am+n + am-n = a + md + nd – d + a + md - nd – d


⇒ am+n + am-n = 2a + 2md – 2d


⇒ am+n + am-n = 2(a + md – d)


⇒ am+n + am-n = 2[a + d(m – 1)]


{∵ an = a + (n – 1)d}


⇒ am+n + am-n = 2am


Hence Proved



Question 5.

Which term of the A.P. 3, 8, 13,… is 248 ?


Answer:

Given A.P is 3, 8, 13,…


Here, a1 = a = 3, a2 = 8


Common difference, d = a2 – a1 = 8 – 3 = 5


We know, an = a + (n – 1)d where a is first term or a1 and d is common difference


∴ an = 3 + (n – 1)5


⇒ an = 3 + 5n – 5


⇒ an = 5n – 2


Now, to find which term of A.P is 248


Put an = 248


∴ 5n – 2 = 248


⇒ 5n = 248 + 2


⇒ 5n = 250



⇒ n = 50


Hence, 50th term of given A.P is 248



Question 6.

Which term of the A.P. 84, 80, 76,… is 0 ?


Answer:

Given A.P is 84, 80, 76,…


Here, a1 = a = 84, a2 = 88


Common difference, d = a2 – a1 = 80 – 84 = -4


We know, an = a + (n – 1)d where a is first term or a1 and d is common difference


∴ an = 84 + (n – 1)-4


⇒ an = 84 – 4n + 4


⇒ an = 88 – 4n


Now, to find which term of A.P is 0


Put an = 0


∴ 88 – 4n = 0


⇒ -4n = -88



⇒ n = 22


Hence, 22th term of given A.P is 0



Question 7.

Which term of the A.P. 4, 9, 14,… is 254 ?


Answer:

Given A.P is 4, 9, 14,…


Here, a1 = a = 4, a2 = 9


Common difference, d = a2 – a1 = 9 – 4 = 5


We know, an = a + (n – 1)d where a is first term or a1 and d is common difference


∴ an = 4 + (n – 1)5


⇒ an = 4 + 5n – 5


⇒ an = 5n – 1


Now, to find which term of A.P is 254


Put an = 254


∴ 5n – 1 = 254


⇒ 5n = 254 + 1


⇒ 5n = 255



⇒ n = 51


Hence, 51st term of given A.P is 254



Question 8.

Is 68 a term of the A.P. 7, 10, 13,…?


Answer:

Given A.P is 7, 10, 13,…


Here, a1 = a = 7, a2 = 10


Common difference, d = a2 – a1 = 10 – 7 = 3


We know, an = a + (n – 1)d where a is first term or a1 and d is common difference and n is any natural number


∴ an = 7 + (n – 1)3


⇒ an = 7 + 3n – 3


⇒ an = 3n + 4


Now, to find whether 68 is a term of this A.P. or not


Put an = 68


∴ 3n + 4 = 68


⇒ 3n = 68 – 4


⇒ 3n = 64



is not a natural number


Hence, 68 is not a term of given A.P.



Question 9.

Is 302 a term of the A.P. 3, 8, 13,…?


Answer:

Given A.P is 3, 8, 13,…


Here, a1 = a = 3, a2 = 8


Common difference, d = a2 – a1 = 8 – 3 = 5


We know, an = a + (n – 1)d where a is first term or a1 and d is common difference and n is any natural number


∴ an = 3 + (n – 1)5


⇒ an = 3 + 5n – 5


⇒ an = 5n – 2


To find: whether 302 is a term of this A.P. or not


Put an = 302


∴ 5n – 2 = 302


⇒ 5n = 302 + 2


⇒ 5n = 304



is not a natural number


Hence,304 is not a term of given A.P.



Question 10.

Which term of the sequence is the first negative term?


Answer:




We know, an = a + (n – 1)d where a is first term or a1 and d is common difference and n is any natural number






Now, to find first negative term


Put an < 0







⇒ n > 33


Hence, 34th term is first negative term of given A.P



Question 11.

Which term of the sequence 12 + 8i, 11 + 6i, 10 + 4i, … is (a) purely real (b) purely imaginary ?


Answer:

Given A.P is 12 + 8i, 11 + 6i, 10 + 4i, …


Here, a1 = a = 12 + 8i, a2 = 11 + 6i


Common difference, d = a2 – a1


= 11 + 6i – (12 + 8i) = 11 – 12 + 6i – 8i = -1 – 2i


We know, an = a + (n – 1)d where a is first term or a1 and d is common difference and n is any natural number


∴ an = 12 + 8i + (n – 1) -1 – 2i


⇒ an = 12 + 8i – n – 2ni + 1 + 2i


⇒ an = 13 + 10i – n – 2ni


⇒ an = (13 – n) + (10 – 2n)i


To find purely real term of this A.P., imaginary part have to be zero


∴ 10 – 2n = 0


⇒ 2n = 10



⇒ n = 5


Hence, 5th term is purely real


To find purely imaginary term of this A.P., real part have to be zero


∴ 13 – n = 0


⇒ n = 13


Hence, 13th term is purely imaginary



Question 12.

How many terms are in A.P. 7, 10, 13,…43?


Answer:

Given A.P is 7, 10, 13,…


Here, a1 = a = 7, a2 = 10


Common difference, d = a2 – a1 = 10 – 7 = 3


We know, an = a + (n – 1)d where a is first term or a1 and d is common difference and n is any natural number


∴ an = 7 + (n – 1)3


⇒ an = 7 + 3n – 3


⇒ an = 3n + 4


To find total terms of the A.P., put an = 43 as 43 is last term of A.P.


∴ 3n + 4 = 43


⇒ 3n = 43 – 4


⇒ 3n = 39



⇒ n = 13


Hence, there are total 13 terms in the given A.P.



Question 13.

How many terms are there in the A.P.


Answer:




We know, an = a + (n – 1)d where a is first term or a1 and d is common difference and n is any natural number






To find total terms of the A.P., put a_nis last term of A.P.






⇒ n = 27


Hence, there are total 27 terms in the given A.P.



Question 14.

The first term of an A.P. is 5, the common difference is 3, and the last term is 80; find the number of terms.


Answer:

Given,


a = 5, last term, l = an = 80


Common difference, d = 3


We know, an = a + (n – 1)d where a is first term or a1 and d is common difference and n is any natural number


∴ an = 5 + (n – 1)3


⇒ an = 5 + 3n – 3


⇒ an = 3n + 2


To find total terms of the A.P., put an = 80 as 80 is last term of A.P.


∴ 3n + 2 = 80


⇒ 3n = 80 – 2


⇒ 3n = 78



⇒ n = 26


Hence, there are total 26 terms in the given A.P.



Question 15.

The 6th and 17th terms of an A.P. are 19 and 41 respectively. Find the 40th term.


Answer:

Given,


6th term of an A.P is 19 and 17th terms of an A.P. is 41


⇒ a6 = 19 and a17 = 41


We know, an = a + (n – 1)d where a is first term or a1 and d is common difference and n is any natural number


When n = 6:


∴ a6 = a + (6 – 1)d


⇒ a6 = a + 5d


Similarly, When n = 17:


∴ a17 = a + (17 – 1)d


⇒ a17 = a + 16d


According to question:


a6 = 19 and a17 = 41


⇒ a + 5d = 19 ………………(i)


And a + 16d = 41…………..(ii)


Subtracting equation (i) from (ii):


a + 16d – (a + 5d) = 41 – 19


⇒ a + 16d – a – 5d = 22


⇒ 11d = 22



⇒ d = 2


Put the value of d in equation (i):


a + 5(2) = 19


⇒ a + 10 = 19


⇒ a = 19 – 10


⇒ a = 9


As, an = a + (n – 1)d


a40 = a + (40 – 1)d


⇒ a40 = a + 39d


Now put the value of a = 9 and d = 2 in a40


⇒ a40 = 9 + 39(2)


⇒ a40 = 9 + 78


⇒ a40 = 87


Hence, 40th term of the given A.P. is 87



Question 16.

If 9th term of an A.P. is Zero, prove that its 29th term is double the 19th term.


Answer:

Given,


9th term of an A.P is 0


⇒ a9 = 0


To prove: a29 = 2a19


We know, an = a + (n – 1)d where a is first term or a1 and d is common difference and n is any natural number


When n = 9:


∴ a9 = a + (9 – 1)d


⇒ a9 = a + 8d


According to question:


a9 = 0


⇒ a + 8d = 0


⇒ a = -8d


When n = 19:


∴ a19 = a + (19 – 1)d


⇒ a19 = a + 18d


⇒ a19 = -8d + 18d


⇒ a19 = 10d


When n = 29:


∴ a29 = a + (29 – 1)d


⇒ a29 = a + 28d


{∵ a = -8d}


⇒ a29 = -8d + 28d


⇒ a29 = 20d


⇒ a29 = 2×10d


{∵ a19 = 10d}


⇒ a29 = 2a19


Hence Proved



Question 17.

If 10 times the 10th term of an A.P. is equal to 15 times the 15th term, show that the 25th term of the A.P. is Zero.


Answer:

Given,


10 times the 10th term of an A.P. is equal to 15 times the 15th term


⇒ 10a10 = 15a15


To prove: a25 = 0


We know, an = a + (n – 1)d where a is first term or a1 and d is common difference and n is any natural number


When n = 10:


∴ a10 = a + (10 – 1)d


⇒ a10 = a + 9d


When n = 15:


∴ a15 = a + (15 – 1)d


⇒ a15 = a + 14d


When n = 25:


∴ a25 = a + (25 – 1)d


⇒ a25 = a + 24d ………(i)


According to question:


10a10 = 15a15


⇒ 10(a + 9d) = 15(a + 14d)


⇒ 10a + 90d = 15a + 210d


⇒ 10a – 15a + 90d – 210d = 0


⇒ -5a – 120d = 0


⇒ -5(a + 24d) = 0


⇒ a + 24d = 0


⇒ a25 = 0 (From (i))


Hence Proved



Question 18.

The 10th and 18th term of an A.P. are 41 and 73 respectively, find 26th term.


Answer:

Given: 10th term of an A.P is 41, and 18th terms of an A.P. is 73


⇒ a10 = 41 and a18 = 73


We know, an = a + (n – 1)d where a is first term or a1 and d is the common difference and n is any natural number


When n = 10:


∴ a10 = a + (10 – 1)d


⇒ a10 = a + 9d


Similarly, When n = 18:


∴ a18 = a + (18 – 1)d


⇒ a18 = a + 17d


According to question:


a10 = 41 and a18 = 73


⇒ a + 9d = 41 ………………(i)


And a + 17d = 73…………..(ii)


Subtracting equation (i) from (ii):


a + 17d – (a + 9d) = 73 – 41


⇒ a + 17d – a – 9d = 32


⇒ 8d = 32



⇒ d = 4


Put the value of d in equation (i):


a + 9(4) = 41


⇒ a + 36 = 41


⇒ a = 41 – 36


⇒ a = 5


As, an = a + (n – 1)d


a26 = a + (26 – 1)d


⇒ a26 = a + 25d


Now put the value of a = 5 and d = 4 in a26


⇒ a26 = 5 + 25(4)


⇒ a26 = 5 + 100


⇒ a26 = 105


Hence, 26th term of the given A.P. is 105



Question 19.

In a certain A.P. the 24th term is twice the 10th term. Prove that the 72nd term is twice the 34th term.


Answer:

Given: 24th term is twice the 10th term


⇒ a24 = 2a10


To prove: a72 = 2a34


We know, an = a + (n – 1)d where a is first term or a1 and d is common difference and n is any natural number


When n = 10:


∴ a10 = a + (10 – 1)d


⇒ a10 = a + 9d


When n = 24:


∴ a24 = a + (24 – 1)d


⇒ a24 = a + 23d


When n = 34:


∴ a34 = a + (34 – 1)d


⇒ a34 = a + 33d ………(i)


When n = 72:


∴ a72 = a + (72 – 1)d


⇒ a72 = a + 71d


According to question:


a24 = 2a10


⇒ a + 23d = 2(a + 9d)


⇒ a + 23d = 2a + 18d


⇒ a – 2a + 23d – 18d = 0


⇒ -a + 5d = 0


⇒ a = 5d


Now, a72 = a + 71d


⇒ a72 = 5d + 71d


⇒ a72 = 76d


⇒ a72 = 10d + 66d


⇒ a72 = 2(5d + 33d)


{∵ a = 5d}


⇒ a72 = 2(a + 33d)


⇒ a72 = 2a34 (From (i))


Hence Proved



Question 20.

If (m + 1)th term of an A.P. is twice the (n + 1)th term, prove that (3m + 1)th term is twice the (m + n + 1)th term.


Answer:

Given: (m + 1)th term of an A.P. is twice the (n + 1)th term


⇒ am+1 = 2an+1


To prove: a3m+1 = 2am+n+1


We know, an = a + (n – 1)d where a is first term or a1 and d is common difference and n is any natural number


When n = m + 1:


∴ am+1 = a + (m + 1 – 1)d


⇒ am+1 = a + md


When n = n + 1:


∴ an+1 = a + (n + 1 – 1)d


⇒ an+1 = a + nd


According to question:


am+1 = 2an+1


⇒ a + md = 2(a + nd)


⇒ a + md = 2a + 2nd


⇒ a – 2a + md – 2nd = 0


⇒ -a + (m – 2n)d = 0


⇒ a = (m – 2n)d………(i)


an = a + (n – 1)d


When n = m + n + 1:


∴ am+n+1 = a + (m + n + 1 – 1)d


⇒ am+n+1 = a + md + nd


⇒ am+n+1 = (m – 2n)d + md + nd……… (From (i))


⇒ am+n+1 = md – 2nd + md + nd


⇒ am+n+1 = 2md – nd………(ii)


When n = 3m + 1:


∴ a3m+1 = a + (3m + 1 – 1)d


⇒ a3m+1 = a + 3md


⇒ a3m+1 = (m – 2n)d + 3md……… (From (i))


⇒ a3m+1 = md – 2nd + 3md


⇒ a3m+1 = 4md – 2nd


⇒ a3m+1 = 2(2md – nd)


⇒ a3m+1 = 2am+n+1…………(From (ii))


Hence Proved



Question 21.

If the nth term of the A.P. 9, 7, 5,… is same as the nth term of the A.P. 15, 12, 9 … Find n.


Answer:

Given: nth term of the A.P. 9, 7, 5,… is same as the nth term of the A.P. 15, 12, 9 …


We know, an = a + (n – 1)d where a is first term or a1 and d is common difference and n is any natural number


Let A.P. 9, 7, 5,… has first term a1 and common difference d1


⇒ a1 = 9 and a2 = 7


Common difference, d1 = a2 – a1 = 7 – 9 = -2


Now, an = a1 + (n – 1)d1


⇒ an = 9 + (n – 1)(-2)


⇒ an = 9 – 2n + 2


⇒ an = 11 – 2n


Let A.P. 15, 12, 9 … has first term a1 and common difference d1


⇒ b1 = 15 and b2 = 12


Common difference, d2 = b2 – b1 = 12 – 15 = -3


Now, bn = b1 + (n – 1)d2


⇒ bn = 15 + (n – 1)(-3)


⇒ bn = 15 – 3n + 3


⇒ bn = 12 – 3n


According to question:


an = bn


⇒ 11 – 2n = 12 – 3n


⇒ 3n – 2n = 12 – 11


⇒ n = 1


Hence, the value of n is 1



Question 22.

Find the 12th term from the end of the following arithmetic progression:

3, 5, 7, 9, … 201


Answer:

A.P is known for Arithmetic Progression whose common difference = an – an-1 where n > 0


a = a1 = 3, a2 = 5, l = 201


Common difference, d = a2 – a1 = 5 – 3 = 2


We know, nth term from end, bn = l – (n – 1)d where l is last term or a1 and d is common difference and n is any natural number


∴ b12 = 201 – (12 – 1)2


⇒ b12 = 201 – 24 + 2


⇒ b12 = 203 – 24


⇒ b12 = 179


Hence, 12thterm from end for the given A.P is 179



Question 23.

Find the 12th term from the end of the following arithmetic progression:

3, 8, 13, … 253


Answer:

A.P is known for Arithmetic Progression whose common difference = an – an-1 where n > 0


a = a1 = 3, a2 = 8, l = 253


Common difference, d = a2 – a1 = 8 – 3 = 5


We know, nth term from end, bn = l – (n – 1)d where l is last term or a1 and d is common difference and n is any natural number


∴ b12 = 253 – (12 – 1)5


⇒ b12 = 253 – 60 + 5


⇒ b12 = 258 – 60


⇒ b12 = 198


Hence, 12thterm from end for the given A.P is 198



Question 24.

Find the 12th term from the end of the following arithmetic progression:

1, 4, 7, 10, … 88


Answer:

A.P is known for Arithmetic Progression whose common difference = an – an-1 where n > 0


a = a1 = 1, a2 = 4, l = 88


Common difference, d = a2 – a1 = 4 – 1 = 3


We know, nth term from end, bn = l – (n – 1)d where l is last term or a1 and d is common difference and n is any natural number


∴ b12 = 88 – (12 – 1)3


⇒ b12 = 88 – 36 + 3


⇒ b12 = 91 – 36


⇒ b12 = 55


Hence, 12thterm from end for the given A.P is 55



Question 25.

The 4th term of an A.P. is three times the first and the 7th term exceeds twice the third term by 1. Find the first term and the common difference.


Answer:

Given,


4th term of an A.P. is three times the first and the 7th term exceeds twice the third term by 1


⇒ a4 = 3a and a7 = 2a3 + 1


We know, an = a + (n – 1)d where a is first term or a1 and d is common difference and n is any natural number


When n = 4:


∴ a4 = a + (4 – 1)d


⇒ a4 = a + 3d


When n = 7:


∴ a7 = a + (7 – 1)d


⇒ a7 = a + 6d


When n = 3:


∴ a3 = a + (3 – 1)d


⇒ a3 = a + 2d


According to question:


a7 = 2a3 + 1


⇒ a + 6d = 2(a + 2d) + 1


⇒ a + 6d = 2a + 4d + 1


⇒ a – 2a + 6d – 4d = 1


⇒ -a + 2d = 1


⇒ a – 2d = -1……(i)


a4 = 3a


⇒ a + 3d = 3a


⇒ 3d = 3a – a


⇒ 3d = 2a



Put this value of d in equation (i):






⇒ a = 3


Now, put this value of a in equation (ii):




⇒ d = 2


Hence, the value of a and d are 3 and 2 respectively



Question 26.

Find the second term and nth term of an A.P. whose 6th term is 12 and 8th term is 22.


Answer:

Given: 6thterm of an A.P is 12 and 8th terms of an A.P. is 22


⇒ a6 = 12 and a8 = 22


We know, an = a + (n – 1)d where a is first term or a1 and d is common difference and n is any natural number


When n = 6:


∴ a6 = a + (6 – 1)d


⇒ a6 = a + 5d


Similarly, When n = 8:


∴ a8 = a + (8 – 1)d


⇒ a8 = a + 7d


According to question:


a6 = 12 and a8 = 22


⇒ a + 5d = 12 ………………(i)


And a + 7d = 22…………..(ii)


Subtracting equation (i) from (ii):


a + 7d – (a + 5d) = 22 – 12


⇒ a + 7d – a – 5d = 10


⇒ 2d = 10



⇒ d = 5


Put the value of d in equation (i):


a + 5(5) = 12


⇒ a + 25 = 12


⇒ a = 12 – 25


⇒ a = -13


As, an = a + (n – 1)d


a2 = a + (2 – 1)d


⇒ a2 = a + d


Now put the value of a = 9 and d = 2 in an and a2


⇒ an = a + (n – 1)d


⇒ an = -13 + (n – 1)5


⇒ an = -13 + 5n – 5


⇒ an = -18 + 5n


a2 = a + d


⇒ a2 = -13 + 5


⇒ a2 = -8


Hence, 2thterm and nth of the given A.P. are -8 and 5n – 18 respectively



Question 27.

How many numbers of two digit are divisible by 3?


Answer:

For finding total two-digit numbers which are divisible by 3, firstly we will make an A.P. of those two-digit numbers which are divisible by 3.


First two digit number which is divisible by 3 is 12


∴ a1 = a = 12


Next two digit number which is divisible by 3 is 15


∴ a2 = 15


Largest two digit number which is divisible by 3 is 99


∴ an = 99


⇒ A.P. is 12, 15,………,99


We know, an = a + (n – 1)d where a is first term or a and d is common difference and n is any natural number


⇒ a1 = 12, a2 = 15 and an = 99


Common difference, d1 = a2 – a1 = 15 – 12 = 3


Now, an = a1 + (n – 1)d


⇒ an = 12 + (n – 1)3


⇒ 99 = 12 + 3n – 3


⇒ 99 = 3n + 9


⇒ 99 – 9 = 3n


⇒ 90 = 3n


⇒ n = 30


Hence, there are total 30 two-digit numbers which are divisible by 3



Question 28.

An A.P. consists of 60 terms. If the first and the last terms be 7 and 125 respectively, find 32nd term.


Answer:

Given: a = 7, n = 60 and l = a60 = 125


We know, an = a + (n – 1)d where a is first term or a1 and d is common difference and n is any natural number


When n = 60:


∴ a60 = a + (60 – 1)d


⇒ a60 = a + 59d


When n = 32:


∴ a32 = a + (32 – 1)d


⇒ a32 = a + 31d …………(i)


According to question:


a60 = 125


⇒ a + 59d = 125


⇒ 7 + 59d = 125


⇒ 59d = 125 – 7


⇒ 59d = 118



⇒ d = 2


Now put the value of a = 7 and d = 2 in a32


⇒ a32 = a + 31d………(From (i))


⇒ a32 = 7 + 31(2)


⇒ a32 = 7 + 62


⇒ a32 = 69


Hence, 32thterm of the given A.P. is 69



Question 29.

The sum of the 4th and the 8th terms of an A.P. is 24, and the sum of the 6th and 10th term is 34. Find the first term and the common difference of the A.P.


Answer:

Given: the sum of the 4th and the 8th terms of an A.P. is 24, and the sum of the 6th and 10th term is 34


⇒ a4 + a8 = 24 and a6 + a10 = 34


We know, an = a + (n – 1)d where a is first term or a1 and d is the common difference and n is any natural number


When n = 4:


∴ a4 = a + (4 – 1)d


⇒ a4 = a + 3d


When n = 6:


∴ a6 = a + (6 – 1)d


⇒ a6 = a + 5d


When n = 8:


∴ a8 = a + (8 – 1)d


⇒ a8 = a + 7d


When n = 10:


∴ a10 = a + (10 – 1)d


⇒ a10 = a + 9d


According to question:


a4 + a8 = 24


⇒ a + 3d + a + 7d = 24


⇒ 2a + 10d = 24


⇒ 2(a + 5d) = 24



⇒ a + 5d = 12………(i)


a6 + a10 = 34


⇒ a + 5d + a + 9d = 34


⇒ 2a + 14d = 34


⇒ 2(a + 7d) = 34



⇒ a + 7d = 17………(ii)


Subtracting equation (i) from (ii):


a + 7d – (a + 5d) = 17 – 12


⇒ a + 7d – a – 5d = 5


⇒ 2d = 5



Put the value of d in equation (i):







Hence, the value of a and d areand respectively



Question 30.

How many numbers are there between 1 and 1000 which when divided by 7 leave remainder 4?


Answer:

For finding total numbers between 1 and 1000 which when divided by 7 leave remainder 4, firstly we will make an A.P. of those numbers which when divided by 7 leave remainder 4.


First number which when divided by 7 leave remainder 4 is 4


∴ a1 = a = 4


Next number which when divided by 7 leave remainder 4 is 11


∴ a2 = 11


Largest three-digit number which when divided by 7 leave remainder 4 is 998


∴ an = 998


⇒ A.P. is 4, 11,………,998


We know, an = a + (n – 1)d where a is first term or a and d is common difference and n is any natural number


⇒ a1 = 4, a2 = 11 and an = 998


Common difference, d1 = a2 – a1 = 11 – 4 = 7


Now, an = a1 + (n – 1)d


⇒ an = 4 + (n – 1)7


⇒ 998 = 4 + 7n – 7


⇒ 998 = 7n – 3


⇒ 998 + 3 = 7n


⇒ 1001 = 7n


⇒ n = 143


Hence, there are total 143 numbers between 1 and 1000 which when divided by 7 leave remainder 4.



Question 31.

The first and the last term of an A.P. are a and l respectively. Show that the sum of the nth term from the beginning and the nth term from the end is a + l.


Answer:

To prove: sum of the nth term from the beginning and the nth term from the end is a + l


We know, an = a + (n – 1)d where a is first term or a1 and d is the common difference, and n is any natural number, and nth term from the end is an = l – (n – 1)d


Now,


an + an = a + (n – 1)d + l – (n – 1)d


⇒ an + an = a + nd – d + l – nd + d


⇒ an + an = a + l


Hence Proved



Question 32.

If an A.P. is such that find .


Answer:

Given:



To find:


We know, an = a + (n – 1)d where a is a first term or a1 and d is the common difference and n is any natural number


When n = 4:


∴ a4 = a + (4 – 1)d


⇒ a4 = a + 3d


When n = 6:


∴ a6 = a + (6 – 1)d


⇒ a6 = a + 5d


When n = 7:


∴ a7 = a + (7 – 1)d


⇒ a7 = a + 6d


When n = 8:


∴ a8 = a + (8 – 1)d


⇒ a8 = a + 7d


According to the question:




⇒ 3(a + 3d) = 2(a + 6d)


⇒ 3a + 9d = 2a + 12d


⇒ 3a – 2a = 12d – 9d


⇒ a = 3d


Now,






Hence, the value of



Question 33.

If θ1, θ2, θ3, …, θn are in AP, whose common difference is d, show that


Answer:

Given: θ1, θ2, θ3,…, θn is A.P


∴ θ2 – θ1 = θ3 – θ2 = θ4 – θ3 =…………= θn – θn-1 = d (Common Difference)


Now,



Multiplying and dividing by sin d:




{∵ sin(A - B) = sin A cos B - cos A sin B}






Similarly,




Take L.H.S.:



Putting the value of (i), (ii) and (iii):





= R.H.S.


Hence Proved




Exercise 19.3
Question 1.

The Sum of the three terms of an A.P. is 21 and the product of the first, and the third terms exceed the second term by 6, find three terms


Answer:

Given: the sum of first three terms is 21


To find: the first three terms of AP


Assume the first three terms are a - d, a, a + d where a is the first term and d is the common difference


So, sum of first three terms is a - d + a + a + d = 21


3a = 21


a = 7


it is also given that product of first and third term exceeds the second by 6


so (a - d)(a + d) - a = 6


a2 - d2 - a = 6


substituting a = 7


72 - d2 - 7 = 6


d2 = 36


d = 6 or d = - 6


Hence the terms of AP are a - d, a, a + d which is 1, 7, 13 or 13, 7, 1



Question 2.

Three numbers are in A.P. If the sum of these numbers be 27 and the product 648, find the numbers


Answer:

Given: sum of first three terms is 27


To find: the first three terms of AP


Assume the first three terms are a - d, a, a + d where a is the first term and d is the common difference


So, sum of first three terms is a - d + a + a + d = 27


3a = 27


a = 9


given that the product of three terms is 648


so a3 - ad2 = 648


substituting a = 9


93 - 9d2 = 648


729 - 9d2 = 648


81 = 9d2


d = 3 or d = - 3


hence the given terms are a - d, a, a + d which is 6, 9, 12 or 12, 9, 6



Question 3.

Find the four numbers in A.P., whose sum is 50 and in which the greatest number is 4 times the least.


Answer:

Given: Sum of four terms is 50


To find: first four terms of AP


Assume these four terms are a - 2d, a - d, a + d, a + 2d


Sum of these terms is 4a = 50



It is also given that the greatest number is 4 time the least


a + 2d = 4(a - 2d)


3a = 10d



Substituting



Hence the terms of AP are a - d, a, a + d which is



Question 4.

The sum of three numbers in A.P. is 12, and the sum of their cubes is 288. Find the numbers.


Answer:

assume the numbers in AP are a - d, a, a + d

Given that the sum of three numbers is 12


To find: the first three terms of AP


So,


3a = 12


a = 4


It is also given that the sum of their cube is 288


(a - d)3 + a3 + (a + d)3 = 288


a3 - d3 - 3ad(a - d) + a3 + a3 + d3 + 3ad(a + d) = 288


substituting a = 4 we get


64 - d3 - 12d(4 - d) + 64 + 64 + d3 + 12d(4 + d) = 288


192 + 24d2 = 288


d = 2 or d = - 2


hence the numbers are a - d, a, a + d which is 2, 4, 6 or 6, 4, 2



Question 5.

If the sum of three numbers in A.P. is 24 and their product is 440, find the numbers.


Answer:

Given: sum of first three terms is 24


To find: the first three terms of AP


Assume the first three terms are a - d, a, a + d where a is the first term and d is the common difference


So, sum of first three terms is a - d + a + a + d = 24


3a = 24


a = 8


given that the product of three terms is 440


so a3 - ad2 = 440


substituting a = 8


83 - 8d2 = 440


512 - 8d2 = 440


72 = 8d2


d = 3 or d = - 3


hence the given terms are a - d, a, a + d which is 5, 8, 11 or 11, 8, 5



Question 6.

The angles of a quadrilateral are in A.P. whose common difference is 10. Find the angles


Answer:

assume the angles are a - 2d, a - d, a + d, a + 2d

We know that the sum of all angles in a quadrilateral is 360


Given: d = 10


To find: angles of the quadrilateral


So, a - 2d + a - d + a + d + a + 2d = 360


4a = 360


a = 90


hence the angles are a - 2d, a - d, a + d, a + 2d which is 70, 80, 100, 110




Exercise 19.4
Question 1.

Find the sum of the following arithmetic progressions:

50, 46, 42, …. To 10 terms


Answer:

for the given AP the first term a is 50, and common difference d is a difference of second term and first term, which is 46 - 50 = - 4

To find: the sum of given AP


The formula for sum of AP is given by



Substituting the values in the above formula



s = 5 × 64


s = 320



Question 2.

Find the sum of the following arithmetic progressions:

1, 3, 5, 7, … to 12 terms


Answer:

for the given AP the first term a is 1, and common difference d is a difference of the second term and first term, which is 3 - 1 = 2

To find: the sum of given AP


The formula for sum of AP is given by



Substituting the values in the above formula



s = 6 × 24


s = 144



Question 3.

Find the sum of the following arithmetic progressions:

3, , 6, , … to 25 terms


Answer:

for the given AP the first term a is 3, and common difference d is a difference of the second term and first term, which is

To find: the sum of given AP


The formula for sum of AP is given by



Substituting the values in the above formula



s = 25 × 21


s = 525



Question 4.

Find the sum of the following arithmetic progressions:

41, 36, 31, … to 12 terms


Answer:

for the given AP the first term a is 41, and common difference d is a difference of the second term and first term, which is 36 - 41 = - 5

To find: the sum of given AP


The formula for sum of AP is given by



Substituting the values in the above formula



s = 6 × 27


s = 162



Question 5.

Find the sum of the following arithmetic progressions:

a + b, a – b, a – 3b, … to 22 terms


Answer:

for the given AP the first term a is a + b, and common difference d is a difference of the second term and the first term, which is a - b - (a + b) = - 2b


To find: the sum of given AP


The formula for sum of AP is given by



Substituting the values in the above formula



s = 11(2a - 40b)



Question 6.

Find the sum of the following arithmetic progressions:
(x-y)2, (x2+y2), (x+y)2, … to n terms


Answer:

for the given AP the first term a is (x - y)2 and common difference d is a difference of the second term and first term, which is

To find: the sum of given AP


Formula: for the sum of AP is given by



Substituting the values in the above formula








Question 7.

Find the sum of the following arithmetic progressions:

, … to n terms


Answer:

for the given AP the first term a is (x - y)2 and common difference d is a difference of the second term and first term, which is

To find: the sum of given AP


The formula for sum of AP is given by



Substituting the values in the above formula




Question 8.

Find the sum of the following series:

2 + 5 + 8 + … + 182


Answer:

for the given AP the first term a is 2, and common difference d is a difference of the second term and first term, which is 5 - 2 = 3

To find: the sum of given AP


The formula for the sum of AP is given by



Substituting the values in the above formula





s = 92 × 61


s = 5612



Question 9.

Find the sum of the following series:

101 + 99 + 97 + … + 47


Answer:

for the given AP the first term a is 101, and common difference d is a difference of second term and first term, which is 99 - 101 = - 2

To find: the sum of given AP


The formula for sum of AP is given by



Substituting the values in the above formula





s = 14 × 148


s = 2072



Question 10.

Find the sum of the following series:

(a - b)2 + (a2 + b2) + (a + b)2 + s…. + [(a + b)2 + 6ab]


Answer:

for the given AP the first term a is (a - b)2 and common difference d is a difference of second term and first term, which is

To find: the sum of given AP


The formula for sum of AP is given by



Substituting the values in the above formula




n = 5




Question 11.

Find the sum of first n natural numbers.


Answer:

for the given AP the first term a is 1, and common difference d is a difference of the second term and first term, which is 2 - 1 = 1

To find: the sum of given AP


The formula for the sum of AP is given by



Substituting the values in the above formula





Question 12.

Find the sum of all - natural numbers between 1 and 100, which are divisible by 2 or 5


Answer:

According to the given question we have to find the sum of natural numbers between 1 and 100 which are divisible by 2 or 5, so the sequence is 2, 4, 6…98 which are multiples of 2 and another sequence 5, 15, 25….95

To find: the sum of all - natural numbers between 1 and 100, which are divisible by 2 or 5


For the first sequence, it is an AP with first term a as 2 and common difference d as 2


Hence the sum is given by the formula





For the second sequence, it is an AP with first term a as 5 and common difference d as 10


Hence the sum is given by the formula




Hence total sum is s1 + s2 = 2450 + 500 = 2950



Question 13.

Find the sum of first n odd natural numbers.


Answer:

given an AP of first n odd natural numbers whose first term a is 1, and common difference d is 3

Given sequence is 1, 3, 5, 7……n


To find: the sum of first n natural numbers


Hence the sum is given by the formula


Substituting the values in the above equation we get



s = n2



Question 14.

Find the sum of all odd numbers between 100 and 200


Answer:

the given sequence is 101, 103, ….199

Given an AP whose first term a is 101, and common difference d is 2


To find: the sum of all odd numbers between 100 and 200


Hence the sum is given by the formula


Substituting the values in the above equation we get



Now for the finding number of terms, the formula is




Substituting n is the sum formula we get



s = 25 × 300


s = 7500



Question 15.

Show that the sum of all odd integers between 1 and 1000 which are divisible by 3 is 83667


Answer:

according to given conditions the sequence is 3, 9, 15, 21….999

Given an AP whose first term is 3 and d is 6. Hence sum is given by


To prove: the sum of all odd integers between 1 and 1000 which are divisible by 3 is 83667


Hence the sum is given by the formula


Now for the finding number of terms, the formula is




n = 167


Substituting n is the sum formula we get



s = 83667


Hence, Proved.



Question 16.

Find the sum of all integers between 84 and 719, which are multiples of 5


Answer:

given an AP is required of all integers between 84 and 719, which are multiples of 5

To find: the sum of all integers between 84 and 719, which are multiples of 5


So, the sequence is 85, 90, 95….715


It is an AP whose first term is 85 and d is 5


Hence the sum is given by the formula


Now for the finding number of terms, the formula is




n = 127


Substituting n is the sum formula we get



s = 50800



Question 17.

Find the sum of all integers between 50 and 500 which are divisible by 7


Answer:

given an AP is required of all integers between 50 and 500, which are multiples of 7

To find: the sum of all integers between 50 and 500 which are divisible by 7


So, the sequence is 56, 63, 70….497


It is an AP whose first term is 56 and d is 7


Hence the sum is given by the formula


Now for the finding number of terms, the formula is




n = 64


Substituting n is the sum formula we get



s = 17696



Question 18.

Find the sum of all even integers between 101 and 999


Answer:

given an AP is required of all integers between 101 and 999 which are even

To find: the sum of all even integers between 101 and 999


So, the sequence is 102, 104, 106….998


It is an AP whose first term is 102 and d is 2


Hence the sum is given by the formula


Now for the finding number of terms, the formula is




n = 449


Substituting n is the sum formula we get



s = 246950



Question 19.

Find the sum of all integers between 100 and 550, which are divisible by 9


Answer:

given an AP is required of all integers between 100 and 550, which are multiples of 9

To find: the sum of all integers between 100 and 550, which are divisible by 9


So, the sequence is 108, 117, 126….549


It is an AP whose first term is 108 and d is 9


Hence the sum is given by the formula


Now for the finding number of terms, the formula is




n = 50


substituting n in the sum formula we get



s = 16425



Question 20.

Find the sum of the series: 3 + 5 + 7 + 6 + 9 + 12 + 9 + 13 + 17 + … to 3n terms


Answer:

given an AP is whose first term is 3 and d is 2

So, the sum is given by formula


To find: sum of the given AP


Substituting the values in the formula




s = 3n(3n + 2)



Question 21.

Find the sum of all those integers between 100 and 800 each of which on division by 16 leaves the remainder 7


Answer:

given an AP is required of all integers between 100 and 800, which on division by 16 leaves remainder 7

To find: the sum of all those integers between 100 and 800 each of which on division by 16 leaves the remainder 7


So, the sequence is 103, 119, 135….791


It is an AP whose first term is 103 and d is 16


Hence the sum is given by the formula


Now for the finding number of terms, the formula is




n = 44


substituting n in the sum formula, we get



s = 19668



Question 22.

Solve:

25 + 22 + 19 + 16 + … + x = 115


Answer:

given is an AP whose first term a is 25 and d is - 3(22 - 25) with a number of terms equal to..

To find: the value of x for the given AP


Now for the finding number of terms, the formula is




Hence the sum is given by the formula


Now substituting the values in the above formula



It is given that s = 115, so equating the above expression to 115



Solving we get n is 10 or


Since n cannot be a fraction, so we choose n as 10



x = - 2



Question 23.

Solve:

1 + 4 + 7 + 10 + …. + x = 590


Answer:

given is an AP whose first term is 1 and d is 3(4 - 1) with a number of terms equal to

To find: the value of x for the given AP


Now for the finding number of terms, the formula is




Hence the sum is given by the formula


Now substituting the values in the above formula



It is given that s = 590, so equating the above expression to 590



Solving we get n is 20 or


Since n cannot be a fraction, so we choose n as 20



x = 58



Question 24.

Find the rthterm of an A.P., the sum of whose first n terms is 3n2 +2n


Answer:

It is given that the sum of n terms is 3n2 + 2n

To find: rth term of an AP, the sum of whose first term is 3n2 + 2n


So, the sum of n - 1 terms is 3(n - 1)2 + 2(n - 1)


formula TN = SN - SN - 1


So, Tr = 3r2 + 2r - [3(r - 1)2 + 2(r - 1)]


Tr = 3r2 + 2r - [3(r2 + 1 - 2r) + 2(r - 1)]


Tr = - 1 + 6r



Question 25.

How many terms are there in the A.P. whose first and fifth terms are - 14 and 2 respectively and the sum of the terms is 40?


Answer:

given the first term is - 14, from which we can say that a = - 14

To find: the number of terms in an AP whose first and fifth terms is - 14 and 2 respectively, and the sum of the terms is 40


Given fifth term is 2, so a + 4d = 2


- 14 + 4d = 2


d = 4


we know that the sum of AP is given by the formula:



now substituting the values in the above equation




4n2 - 32n - 80 = 0


n2 - 8n - 20 = 0


solving we get n as 10 or - 2


Since the number of terms cannot be negative hence n is 10



Question 26.

The sum of the first 7 terms of an A.P. is 10, and that of the next 7 terms is 17. Find the progression


Answer:

Assuming the first term as a and common difference as d

To find: the progression


So, the sum of first 7 terms is given by


a + a + d + a + 2d + a + 3d…. a + 6d = 10


7a + 21d = 10…(i)


In the second part it is given that sum of next seven terms is 17


a + 7d + a + 8d + a + 9d…. a + 13d = 7


7a + 70d = 7…(ii)


Solving (i) and (ii) we get


10 - 21d = 7 - 70d


3 = - 49d




Hence the sequence is given by a, a + d, a + 2d…. which is




Question 27.

The third term of an A.P. is 7, and the seventh term exceeds 3 times the third term by 2. Find the first term, the common difference and the sum of the first 20 terms.


Answer:

given third term of AP is 7

So, a + 2d = 7…(i)


It is also given that the seventh term exceeds 3 times the third term by 2


To find: first term, the common difference and the sum of the first 20 terms


a + 6d - 3(a + 2d) = 2


- 2a = 2


a = - 1


d = 4


we know that the sum of AP is given by the formula:



substituting the values in above equation



s = 740



Question 28.

The first term of an A.P. is 2, and the last term is 50. The sum of all these terms is 442. Find the common difference.


Answer:

given a1 = 2 and an = 50

To find: the common difference


we know that the sum of AP is given by the formula:



substituting the values in above equation we get



n = 17


Now for the finding number of terms, the formula is



we know that


d = 3



Question 29.

The number of terms of an A.P. is even; the sum of the odd term is 24, of the even terms is 30, and the last term exceeds the first by , find the number of terms and the series


Answer:

let the number of terms be 2n with first term a and common difference d

To find: number of terms and the series


The last term is given as a + (2n - 1) d


It is given that the last term exceeds the first by


So


…(i)


Sum of odd terms is 24


…(ii)


Sum of even terms is 30


…(iii)


Solving (i), (ii) and (iii) we get


d = 1.5


substituting d in (i) we get n = 4


and substituting d and n in (ii) we get a = 1.5


so, the number of terms is 2n = 8, and the sequence is given by a, a + d, a + 2d….


1.5, 3, 4.5, 6, 7.5 , 9, 10.5 , 12



Question 30.

If Sn = n2p and in an A.P., prove that Sp = p3


Answer:

given an AP whose sum of n terms is n2p and same AP with m terms whose sum is m2p

To prove: Sp = p3


we know that the sum of AP is given by the formula:



substituting the values in the above equation, we get


…. (i)


Similarly, for series with m terms


…(ii)


Subtracting (ii) from (i) we get


d = 2p


substituting d in (i) we get


a = p


Now using the sum formula for AP consisting of p terms we get



Substituting the values in the above equation



SP = p3



Question 31.

If the 12th term of an A.P. is - 13 and the sum of the first 4 terms is 24, what is the sum of the first 10 terms?


Answer:

assuming an AP whose first term is a and the common difference is d

To find: the sum of first 10 terms


Given 12th term is - 13


So, a + 11d = - 13...(i)


Given the sum of first four terms is 24


So 4a + 6d = 24…(ii)


Solving (i) and (ii) we get


a = 9 and d = - 2


we know that the sum of AP is given by the formula:



substituting the values in the above equation



s = 0



Question 32.

If the 5th and 12th terms of an A.P. are 30 and 65 respectively, what is the sum of the first 20 terms?


Answer:

assuming an AP whose first term is a and the common difference is d

To find: the sum of the first 20 terms


Given 5th term is 30


So, a + 4d = 30...(i)


Given 12th term is 65


So, a + 11d = 65…(ii)


Solving (i) and (ii) we get


a = 10 and d = 5


we know that the sum of AP is given by the formula:



substituting the values in the above equation



s = 1150



Question 33.

Find the sum of n terms of the A.P. whose kth terms is 5k + 1


Answer:

given an AP whose kth term is 5k + 1

To find: the sum of n terms of an AP whose kth term is 5k + 1


So substituting k = 1 to get the first term a1 = 6


Substituting k = 2 to get the second term a2 = 11


d = a2 - a1 = 5


we know that the sum of AP is given by the formula:



substituting the values in the above equation





Question 34.

Find the sum of all two digit numbers which when divided by 4, yields 1 as the remainder.


Answer:

the series which satisfies the above condition is

13, 17, 21….97


To find: the sum of all two - digit numbers which when divided by 4, yields 1 as the remainder


So, it is an AP whose first term is 13 and common difference d as 4


Now for the finding number of terms, the formula is



And


n = 22


we know that the sum of AP is given by the formula:



substituting the values in the above equation



s = 1210



Question 35.

If the sum of a certain number of terms of the A.P. 25, 22, 19, …. is 116. Find the last term


Answer:

given an AP whose first term is 25 and the common difference is - 3

To find: the last term of a given AP


we know that the sum of AP is given by the formula




Solving we get


3n2 - 53n + 232 = 0


n = 8 or


n cannot be a fraction hence we choose n as 8


Now for the finding number of terms, the formula is




solving we get the last term as 4



Question 36.

Find the sum of odd integers from 1 to 2001.


Answer:

given is an AP whose first term is 1 and d is 2

To find: the sum of odd integers from 1 to 2001


Now for the finding number of terms, the formula is




n = 1001


we know that the sum of AP is given by the formula:



substituting the values in the above equation



s = 1002001



Question 37.

How many terms of the A.P. - 6, - , - 5, …, are needed to give the sum - 25?


Answer:

given is an AP whose first term is - 6 and d is

To find: the number of terms required to make the sum of a given AP to - 25


we know that the sum of AP is given by the formula:



substituting the values in the above equation




100 = - 25n + n2


n2 - 25n + 100 = 0


(n - 20) (n - 5) = 0


solving we get n = 20 or n = 5



Question 38.

In an A.P. the first term is 2, and the sum of the first 5 terms is one-fourth of the next 5 terms. Show that 20th term is - 112


Answer:

given an AP whose first term is 2

To find: 20th term of an AP


It is given that the sum of the first five terms is one-fourth of the next 5 terms


So


20a + 40d = 5a + 35d


15a + 5d = 0


Substituting a as 2


We get d = - 6


Hence 20th term is a + 19d = - 112



Question 39.

If S1 be the sum of (2n + 1) terms of an A.P. and S2 sum of its odd terms, then prove that: S1: S2 = (2n + 1) : (n + 1).


Answer:

To prove: S1: S2 = (2n + 1) : (n + 1)

we know that the sum of AP is given by the formula:



Substituting the values in the above equation



For the sum of odd terms, it is given by


s2 = a1 + a3 + a5 + …. a2n + 1


s2 = a + a + 2d + a + 4d + … + a + 2nd


s2 = (n + 1)a + n(n + 1)d


s2 = (n + 1) (a + nd)


Hence s1:s2



Question 40.

Find an A.P. in which the sum of any number of terms is always three times the squared number of these terms


Answer:

To find: AP with given conditions

Given that sum of n terms is 3n2


Sn = 3n2


Similarly, sum of n - 1 terms is 3(n - 1)2


Sn - 1 = 3(n - 1)2


formula TN = Sn - Sn - 1 = 3n2 - 3(n - 1)2


Now substituting n = 1 to get the first term


a1 = 3


Now substituting n = 2 to get the second term


a2 = 9


d = a2 - a1 = 6


hence the series is given by a, a + d, a + 2d…which is 3, 9, 15, 21….



Question 41.

If the sum of n terms of an A.P. is nP + n(n - 1)Q where P and Q are constants, find the common difference.


Answer:

it is given that sum of n terms of AP is

To find: the common difference


Substituting n = 1 gives the sum of the first term, that is the first term itself


a1 = P


Substituting n = 2 gives the sum of first two terms


a1 + a2 = 2P + Q


a2 = P + Q


Now common difference d = a2 - a1 = Q



Question 42.

The sums of n terms of two arithmetic progressions are in the ratio 5n + 4: 9n + 6. Find the ratio of their 18th terms


Answer:

assuming first AP whose first term is a and common difference d

To find: the ratio of 18th term of a given AP


we know that the sum of AP is given by the formula:



Substituting the values in the above equation



assuming second AP whose first term is A and common difference D


we know that the sum of AP is given by the formula:



substituting the values in the above equation




…. (i)


Given the ratio of the 18th term is required of both AP’s which is



Substituting n = 35 in (i) we get




Question 43.

The sum of first n terms of two A.P.’s is in the ratio (7n + 2) : (n + 4). Find the ratio of their 5th terms.


Answer:

assuming first AP whose first term is a and common difference d

To find: the ratio of 5th term of a given AP


we know that the sum of AP is given by the formula:



substituting the values in the above equation



Assuming second AP whose first term is A and common difference D


we know that the sum of AP is given by the formula:



substituting the values in the above equation




…. (i)


Given the ratio of the 5th term is required of both AP’s which is



Substituting n = 9 in (i) we get





Exercise 19.5
Question 1.

If are in A.P., prove that:

are in A.P.


Answer:

We know, if a, b, c are in AP,


Then b - a = c - b


It is said that, are in AP,


Hence,


If are in AP, then



Taking LCM,



Now, LHS =


Multiply c in both numerator and denominator,


We get,


RHS


Multiply a in both numerator and denominator,


We get, =


Therefore, LHS = RHS


=


c(b - a) = a(b-c)


Also,


c(b - a) = a(b - c)


Hence given terms are in AP



Question 2.

If are in A.P., prove that:

a(b + c), b(c + a), c(a + b) are in A.P.


Answer:

It is said that, are in AP,


Hence,


Taking LCM,


.


Multiply in both denominator and numerator with c in LHS and a in RHS



Since,


a(b + c), b(c + a), c(a + b) are to be proved in A.P.


b(c + a)-a(b + c) = c(a + b)-b(c + a)


bc + ba – ab - ca- = ca + ab – bc - ba


cb - ca = ca - ab


c(b - a) = a(c - b)


Hence, given terms are in AP.



Question 3.

If a2, b2, c2 are in A.P., prove that are in A.P.


Answer:

If a2, b2, c2 are in A.P then, b2 - a2 = c2 - b2


If are in A.P. then,





Since, b2 - a2 = c2 - b2


Put b2 - a2= c2 - b2 in above,


We get LHS = RHS


Hence given terms are in AP



Question 4.

If a, b, c are in A.P., then show that:

(i) a2(b + c), b2(c + a), c2(a + b) are also in A.P.

(ii) b + c - a, c + a - b, a + b - c are in A.P.

(iii) bc – a2, ca – b2, ab – c2 are in A.P.


Answer:

(i)


b2c + b2a – a2b – a2c = c2a + c2b – b2a – b2c


c(b2 - a2 ) + ab(b - a) = a(c2 - b2 ) + bc(c - b)


(b - a)(ab + bc + ca) = (c - b)(ab + bc + ca)


b - a = c - b


And since a, b, c are in AP,


b - a = c - b


Hence given terms are in AP


(ii) b + c - a, c + a - b, a + b - c are in A.P.


Therefore, (c + a - b) - (b + c - a) = (a + b - c) - (c + a - b)


2a - 2b = 2b - 2c


b - a = c - b


And since a, b, c are in AP,


b - a = c - b


Hence given terms are in AP.


(iii) bc – a2, ca – b2, ab – c2 are in A.P.


(ca - b2) – (bc - a2) = (ab - c2) – (ca - b2)


(a - b)(a + b + c) = (b - c)(a + b + c)


a - b = b - c


b - a = c - b


And since a, b, c are in AP,


b - a = c - b


Hence given terms are in AP



Question 5.

If are in A.P., prove that:

are in A.P.


Answer:

Since, are in AP,






a(b - c) = c(a - b)


Since, are in A.P.




Multiply in both denominator and numerator with,


C in LHS and a in RHS



Hence given terms are in AP



Question 6.

If are in A.P., prove that:

bc, ca, ab are in A.P.


Answer:

Since, bc, ca, ab are in A.P.


=


=


Since, are in A.P.




Multiply in both denominator and numerator with,


C in LHS and a in RHS



Hence given terms are in AP



Question 7.

If a, b, c are in A.P., prove that:

(a - c)2 = 4 (a - b) (b - c)


Answer:

(a - c)2 = 4 (a - b) (b - c)


a2 + c2 - 2ac = 4(ab – ac – b2 + bc)


a2 + 4c2b2 + 2ac - 4ab - 4bc = 0


(a + c - 2b)2 = 0


a + c - 2b = 0


Since a, b, c are in AP


b - a = c - b


a + c - 2b = 0


Hence,


(a - c)2 = 4 (a - b) (b - c)



Question 8.

If a, b, c are in A.P., prove that:

a2 + c2 + 4ac = 2 (ab + bc + ca)


Answer:

a2 + c2 + 4ac = 2 (ab + bc + ca)


a2 + c2 + 2ac - 2ab - 2bc = 0


(a + c - b)2 – b2 = 0


a + c - b = b


a + c - 2b = 0


Since a, b, c are in AP


b - a = c - b


a + c - 2b = 0


Hence proved



Question 9.

If a, b, c are in A.P., prove that:

a3 + c3 + 6abc = 8b3


Answer:

a3 + c3 + 6abc = 8b3


a3 + c3 - (2b)3 + 6abc = 0


a3 + (-2b)3 + c3 + 3a(-2b)c = 0


Since, if a + b + c = 0, a3 + b3 + c3 = 3abc


(a - 2b + c)3 = 0


a - 2b + c = 0


Since a, b, c are in AP


b - a = c - b


= a + c - 2b = 0


Hence proved



Question 10.

If are in A.P., prove that a, b, c are in A.P.


Answer:

Since are in A.P


Also are also in AP


Therefore,


= are in AP


= are in AP


Multiply by abc in numerator in all terms,


= are in AP


= a, b, c are in AP


Hence proved



Question 11.

Show that x2 + xy + y2, z2 + zx + x2 and y2 + yz + z2 are in consecutive terms of an A.P., if x, y and z are in A.P.


Answer:

Since, x2 + xy + y2, z2 + zx + x2 and y2 + yz + z2 are in AP


(z2 + zx + x2) - (x2 + xy + y2) = (y2 + yz + z2) - (z2 + zx + x2)


Let d = common difference,


Sicex, y, z are in AP


Y = x + d and x = x + 2d


Therefore the above equation becomes,


= (x + 2d)2 + (x + 2d)x - x(x + d) - (x + d)2 = (x + d)2 + (x + d)(x + 2d) - (x + 2d)x – x2


= x2 + 4xd + 4d2 + x2 + 2xd – x2 – xd – x2 - 2xd – d2 = x2 + 2dx + d2 + x2 + 2dx + xd + 2d2 – x2 - 2dx – x2



Hence, proved.


x2 + xy + y2, z2 + zx + x2 and y2 + yz + z2 are in consecutive terms of an A.P.




Exercise 19.6
Question 1.

Find the A.M. between:

(i) 7 and 13 (ii) 12 and - 8 (iii) (x - y) and (x + y)


Answer:

(i) Let A be the Arithmetic mean


Then 7, A, 13 are in AP


A-7 = 13-A


2A = 13 + 7


A = 10


(ii) Let A be the Arithmetic mean


Then 12, A, - 8 are in AP


A - 12 = - 8 - A


2A = 12 + 8


A = 2


(iii) Let A be the Arithmetic mean


Then x - y, A, x + y are in AP


A - (x - y) = (x + y) – A


2A = x + y + x - y


A = x



Question 2.

Insert 4 A.M.s between 4 and 19.


Answer:

Let A1, A2, A3, A4 be the 4 AM Between 4 And 19


Then, 4, A1, A2, A3, A4, 19 are in AP.


We know,





d = 3


Hence,


A1 = a + d = 4 + 3 = 7


A2 = A1 + d = 7 + 3 = 10


A3 = A2 + d = 10 + 3 = 13


A4 = A3 + d = 13 + 3 = 16


Question 3.

Insert 7 A.M.s between 2 and 17.


Answer:

Let A1, A2, A3, A4, A5, A6, A7 be the 7 AMs Between 2 And 17


Then, 2, A1, A2, A3, A4, A5, A6, A7, 17 are in AP


We know,


An = a + (n - 1)d


A9 = 17 = 2 + (9 - 1)d



Therefore,










So, 7 AMs between 2 and 7 are


Question 4.

Insert six A.M.s between 15 and - 13.


Answer:

Let A1, A2, A3, A4, A5, A6 be the 7 AM Between 15 And - 13


Then, 15, A1, A2, A3, A4, A5, A6, - 13 are in AP


We know,


An = a + (n - 1)d


A8 = - 13 = 15 + (8 - 1)d


d = -4


Hence,


A1 = a + d = 15 - 4 = 11


A2 = A1 + d = 11 - 4 = 7


A3 = A2 + d = 7 - 4 = 3


A4 = A3 + d = 3 - 4 = -1


A5 = A4 + d = -1 - 4 = -5


A6 = A5 + d = -5 - 4 = -9



Question 5.

There are n A.M.s between 3 and 17. The ratio of the last mean to the first mean is 3 : 1. Find the value of n.


Answer:

Let the series be 3, A1, A2, A3, ........, An, 17


From the data,


We know total terms in AP are n + 2


So 17 is the (n + 2)th term


We know,


An = a + (n - 1)d


So, 17 = 3 + (n + 2 - 1)d



Therefore,



And



Since,




9n + 51 = 17n + 3


8n = 48



n = 6


There are 6 terms in AP



Question 6.

Insert A.M.s between 7 and 71 in such a way that the 5th A.M. is 27. Find the number of A.M.s.


Answer:

Let the series be 7, A1, A2, A3, ........, An, 71


We know total terms in AP are n + 2


So 71 is the (n + 2)th term


We know,


An = a + (n - 1)d


So, A6 = a + (6 - 1)d


a + 5d = 27 (5th term)


d = 4


71 = (n + 2)th term


71 = a + (n + 2 - 1)d


71 = 7 + n(4)


n = 15


There are 15 terms in AP



Question 7.

If n A.M.s are inserted between two numbers, prove that the sum of the means equidistant from the beginning and the end is constant.


Answer:

Let a and b be the first and last terms and


The series be a, A1, A2, A3, ........, An, b


So We know, Mean


Mean of A1 and An =


A1 = a + d


An = a - d


Therefore, AM


AM between A2 and An-1


Similarly, it is (a + b)/2 for all such numbers, which is constant


Hence, AM = (a + b)/2



Question 8.

If x, y, z are in A.P. and A1is the A.M. of x and y, and A2 is the A.M. of y and z, then prove that the A.M. of A1 and A2 is y.


Answer:

Given that,


A1 = AM of x and y


And A2 = AM of y and z


So,



AM of A1 and A2




Since x, y, z are in AP,


Finally, AM =


Hence proved.



Question 9.

Insert five numbers between 8 and 26 such that the resulting sequence is an A.P


Answer:

Let A1, A2, A3, A4, A5 be the 5 nos Between 8 And 26


Then, 8, A1, A2, A3, A4, A5, 26 are in AP


We know,


An = a + (n - 1)d


A7 = 26 = 8 + (7 - 1)d


d = 3


Hence,


A1 = a + d = 8 + 3 = 11


A2 = A1 + d = 11 + 3 = 14


A3 = A2 + d = 14 + 3 = 17


A4 = A3 + d = 17 + 3 = 20


A5 = A4 + d = 20 + 3 = 23




Exercise 19.7
Question 1.

A man saved ₹ 16500/- in ten years . in each year after the first he saved ₹ 100/- more then he did in the preceding year. How much did he saved in the first year?


Answer:

Given: A man saved 16500/- in ten years


To find: His saving in the first year


Let he saved Rs. x in the first year


Since each year after the first he saved 100/- more then he did in the preceding year


So,


A.P will be x, 100 + x, 200 + x………………..


where x is first term and


common difference, d = 100 + x – x = 100


We know,


Sn is the sum of n terms of an A.P


Formula used:



where a is first term, d is common difference and n is number of terms in an A.P.


According to the question:


Sn = 16500 and n = 10


Therefore,



⇒ 16500 = 5{2x + 9(100)}


⇒ 16500 = 5(2x + 900)


⇒ 16500 = 10x + 4500


⇒ -10x = 4500 – 16500


⇒ –10x = –12000



⇒ x = 1200


Hence, his saving in first year is 1200



Question 2.

A man saves ₹ 32 during the first year, ₹ 36 in the second year and in this way he increases his savings by ₹4 every year. Find in what time his saving will be ₹200.


Answer:

Given: A man saves Rs. 32 during first year and increases his savings by Rs. 4 every year. His total saving is Rs. 200


To find: Time taken in years by him to save Rs. 200.


A man saves in the first year is 32Rs


He saves in the second year is 36Rs.


In this process he increases his savings by Rs. 4 every year


Therefore,


A.P. will be 32, 36, 40,…………………


where 32 is first term and


common difference, d = 36 – 32 = 4


We know,


Sn is the sum of n terms of an A.P


Formula used:



where a is first term, d is common difference and n is number of terms in an A.P.


According to the question:


Sn = 200


Therefore,












n can not be a negative values as days can not hold negative values


⇒ n = 5


Hence, he has to do saving for 5 days to save Rs. 200



Question 3.

A man arranges to pay off a debt of ₹ 3600 by 40 annual instalments which form an arithmetic series. When 30 of the instalments are paid, he dies leaving one-third of the debt unpaid, find the value of the instalment.


Answer:

Given: Total debt is Rs. 3600 and total number of installments are 40 and in 30 installments, he has paid two-third of the debt and dies leaving one-third of amount


Let first instalment be “a”


Let the common difference of the instalments be “d”


Here, S40 = 3600 and n = 40


Formula used:



where a is first term, d is common difference and n is number of terms in an A.P.


Therefore,





⇒ 2a+39d = 180 ………(1)


Since 30 installments are paid and one third of the debt unpaid



So,


S30 = 2400 and n = 30







⇒ 2a+29d = 160 ………(2)


Solving (1) and(2) by substitution method,


2a + 39d = 180


⇒ 2a = 180 – 39d ……(3)


Put value of 2a from (3) in (2):


2a + 29d = 160


⇒ 180 – 39d + 29d = 160


⇒ –10d = 160 – 180


⇒ –10d = –20



⇒ d = 2


Put this value of d in (3):


2a = 180 – 39d


⇒ 2a = 180 – 39(2)


⇒ 2a = 180 – 78


⇒ 2a = 102



⇒ a = 51


Hence, value of first installment = a = Rs. 51



Question 4.

A manufacturer of the radio sets produced 600 units in the third year and 700 units in the seventh year. Assuming that the product increases uniformly by a fixed number every year, find

(і) the production in the first year

(іі) the total product in the 7 years and

(ііі) the product in the 10th year.

Solution |||

(і) the production in the first year

Answer:

Given: 600 and 700 radio sets units are produced in third and seventh year respectively

To find: the production in the first year i.e. a

⇒ a3 = 600 and a7 = 700

Formula used:

For an A.P., an is nth term which is given by,

an = a + (n – 1)d

where a is first term, d is common difference and n is number of terms in an A.P.

Therefore,

a3 = a + (3 – 1)d

⇒ 600 = a + 2d……………………(1)

a7 = a + (7 – 1)d

⇒ 700 = a + 6d

⇒ a = 700 – 6d………………………(2)

Now put this value of a in equation (1):

⇒ 600 = 700 – 6d + 2d

⇒ 600 – 700 = – 6d + 2d

⇒ –100 = –4d



⇒ d = 25

Put d = 25 in equation (2):

⇒ a = 700 – 6(25)

⇒ a = 700 – 150

⇒ a = 550

Production in the first year = a = 550

(іі) the total product in the 7 years


Answer:

To find: the sum of totals products in 7 years i.e. S7


Formula used:



where a is first term, d is common difference and n is number of terms in an A.P.


Therefore,






⇒ S7 = 7(625)


⇒ S7 = 4375


Hence, the total product in the 7 years are 4375


(ііі) the product in the 10th year


Answer:


To find: the product in the 10th year i.e. a10


Formula used:


For an A.P., an is nth term which is given by,


an = a + (n – 1)d


where a is first term, d is common difference and n is number of terms in an A.P.


Therefore,


a10 = 550 + (10 – 1)25


⇒ a10 = 550 + (9)25


⇒ a10 = 550 + 225


⇒ a10 = 775


Hence, the product in the 10th year are 775 units



Question 5.

There are 25 trees at equal distances of 5 meters in a line with a well, the distance of well from the nearest tree being 10 meters. A gardener waters all the trees separately starting from the well and he returns to the well after watering each tree to get water for the next. Find the total distance the gardener will cover in order to water all the trees.


Answer:

Given: total trees are 25 and equal distance between two adjacent trees are 5 meters


To find: the total distance the gardener will cover


As gardener is coming back to well after watering every tree


Distance covered by him to water 1st tree and return to the initial position is 10m + 10m = 20m


Now, distance between adjacent trees is 5m


Distance covered by him to water 2nd tree and return to the initial position is 15m + 15m = 30m


Distance covered by the gardener to water 3rd tree return to the initial postion is 20m + 20m = 40m


Hence distance covered by the gardener to water the trees are in A.P


A.P. is 20, 30, 40 ………upto 25 terms


Here first term,a = 20,common difference, d = 30 – 20 = 10


And n = 25


We need to find S25 which will be the total distance covered by gardener to water 25 trees


Formula used:



where a is first term, d is common difference and n is number of terms in an A.P.


Therefore,






⇒ S25 = 25(140)


⇒ S25 = 3500


Hence, total distance covered by gardener to water trees is 3500 m



Question 6.

A man is employed to count ₹ 10710. He counts at the rate of ₹ 180 per minute for half an hour. After this he counts at the rate of ₹ 3 less every minute then the preceding minute. Find the time taken by him to count the entire amount.


Answer:

Given: Amount to be counted is Rs. 10710


To find: Time taken by man to count the entire amount


He counts at the rate of Rs. 180 per minute for half an hour or 30 minutes


Amount to be counted in an hour = 180 * 30 = Rs. 5400


Amount to be left = 10710 – 5400 = 5310


⇒ Sn = 5310


After an hour, rate of counting is decreasing at Rs. 3 per minute. This rate will form an A.P.


A.P. will be 177, 174, 171,……………………


Here a = 177 and d = 174 – 177 = –3


Formula used:



where a is first term, d is common difference and n is number of terms in an A.P.


Therefore,






⇒ 10620 = 3n(119 – n)




⇒ n2 – 59n – 60n + 3540 = 0


⇒ n(n – 59) – 60(n – 59) = 0


⇒ (n – 59)(n – 60) = 0


⇒ n = 59 or 60


We will take value of n = 59 as at 60th min he will count Rs.0


Therefore, total time taken by him to count the entire amount = 30 + 59 = 89 minutes



Question 7.

A piece of equipment cost a certain factory ₹ 600, 000. If it depreciates in value 15% the first, 13.5% the next year, 12% the third year, and so on. What will be its value at the end of 10 years, all percentages applying to the original cost?


Answer:

Given: A piece of equipment cost a certain factory is 600,000


To find: Value of the equipment at the end of 10 years


It depreciates 15%, 13.5%, 12% in 1st, 2nd, 3rd year and so on.


This means price of equipment is depreciating in an A.P.


A.P. will be 15, 13.5, 12,…………………………upto 10 terms


Hence a = 15, d = 13.5 – 15 = –1.5


Formula used:



where a is first term, d is common difference and n is number of terms in an A.P.


Therefore,


Total percentage of depreciation in 10 years,






⇒ S10 = 82.5


Value of the equipment at the end of 10 years,





= 175 × 600


= 105000


Hence, value of equipment at the end of 10 years is Rs. 105000



Question 8.

A farmer buys a used tractor for ₹ 12000. He pays ₹ 6000 cash and agrees to pay the balance in annual instalments of ₹ 500 plus 12% interest on the unpaid amount. How much the tractor cost him?


Answer:

Given: Price of the tractor is Rs. 12000. He pays ₹ 6000 cash and agrees to pay the balance in annual instalments of ₹ 500 plus 12% interest on the unpaid amount.


To find: Total cost of the tractor if he buys it at installment


Total price = 12000


Paid amount = 6000


Unpaid amount = 12000 – 6000 = 6000


He pays remaining 6000 in n number of installments of 500 each



Cost incurred by him to pay remaining 6000,




Formula used:



where a is first term, d is common difference and n is number of terms in an A.P.


Therefore,







= 10680


Total cost = 6000 + 10680 = 16680


Hence, total cost of the tractor if he buys it at installment is Rs. 16680



Question 9.

Shamshad Ali buys a Scooter for ₹ 22000. He pays ₹ 4000 cash and agrees to pay the balance in annual instalments of ₹ 1000 plus 10% interest on the unpaid amount. How much the Scooter will cost him?


Answer:

Given: Price of the scooter is Rs. 22000. He pays ₹ 4000 cash and agrees to pay the balance in annual instalments of Rs. 1000 plus 10% interest on the unpaid amount.


To find: Total cost of the scooter if he buys it at installment


Total price = 22000


Paid amount = 4000


Unpaid amount = 22000 – 4000 = 18000


He pays remaining 18000 in n number of installments of 1000 each



Cost incurred by him to pay remaining 18000,




Formula used:



where a is first term, d is common difference and n is number of terms in an A.P.


Therefore,







= 35100


Total cost = 4000 + 35100 = 39100


Hence, total cost of the scooter if he buys it at installment is Rs. 39100



Question 10.

The income of a person is ₹ 300, 000 in the first year and he receives an increase of ₹ 10, 000 to his income per year for the next 19 years. Find the total amount, he received in 20 years.


Answer:

Given: The income of a person is ₹ 300, 000 in the first year


and he receives an increase of ₹ 10, 000 to his income per year for the next 19 years.


To find: the total amount he received in 20 years i.e. S20


According to question:


a = 300000, d = 10000 and n = 20


Formula used:



where a is first term, d is common difference and n is number of terms in an A.P.


Therefore,





⇒ S20 = 10(790000)


⇒ S20 = 7900000


Hence, the total amount he received at the end of 20 years is Rs. 790000



Question 11.

A man starts repaying a loan as first instalment of Rs.100. If he increases the instalments by Rs .5 every month, what amount he will pay in 30th instalment?


Answer:

Given: First installment is Rs. 100 and he increases it by Rs. 5 every month


To find: Total amount in 30th installment i.e. a30


According to question:


A.P. will be 100, 105,……………………


a = 100 and d = 5


Formula used:


an = a + (n – 1)d


⇒ a30 = 100 + (30 – 1)5


⇒ a30 = 100 + (29)5


⇒ a30 = 100 + 145


⇒ a30 = 245


Hence, he will pay Rs. 245 in 30th installment



Question 12.

A carpenter was hired to build 192 window frames. The first day he made five frames and each day there after he made two more frames than he made the day before. How many days did it take him to finish the job? [NCERT EXEMPLAR]


Answer:

Given: Carpenter was hired to build 192 frames. He made 5 frames on first day and two more frames for every next day


To find: Number of days taken by him to build 192 frames


According to question:


A.P. will be 5, 7, 9,………………………………


⇒ Sn = 192, a = 5 and d = 7 – 5 = 2


Formula used:



where a is first term, d is common difference and n is number of terms in an A.P.


Therefore,








⇒ n2 + 16n – 12n + 192 = 0


⇒ n(n + 16) – 12(n + 16) = 0


⇒ (n + 16)(n – 12) = 0


⇒ n = –16 or 12


n is number of days and it cannot be negative


Therefore, we will take value of n = 12


Hence, he will take 12 days to build 192 frames



Question 13.

We know that the sum of interior angles of a triangle is 180°. Show that the sums of interior angles of polygons with 3, 4, 5, 6 … sides from an arithmetic progression. Find the sum of interior angles for a 21 sided polygon. [NCERT EXEMPLAR]


Answer:

Given: the sum of interior angles of a triangle is 180°


To prove: Sum of interior angles of polygons with 3, 4, 5,…… forms an A.P.


To find: Sum of interior angles for a 21 sided polygon i.e. a21


We know,


The sum of interior angles of a polygon with n sides,


an = 180° × (n – 2)


Sum of interior angles of a polygon of 3 sides,


a3= 180° × (3 – 2) = 180° × 1 = 180°


Sum of interior angles of a polygon of 4 sides,


a4= 180° × (4 – 2) = 180° × 2 = 360°


Sum of interior angles of a polygon of 5 sides,


a5= 180° × (5 – 2) = 180° × 3 = 540°


A.P is known for Arithmetic Progression whose common difference, d = an – an-1 where n > 0


Here,


d = a4 – a3 = 360° - 180° = 180°


d = a5 – a4 = 540° - 360° = 180°


Common difference is same in both the cases


This shows that sum of interior angles of polygons with 3, 4, 5,…… forms an A.P.


Hence Proved


Sum of interior angles of a polygon of 21 sides,


a21= 180° × (21 – 2) = 180° × 19 = 3420°



Question 14.

In a potato race 20 potato are placed in a line at intervals of 4 meters with the first potato 24 meters from the starting point. A contestant is required to bring the potatoes back to the starting place one at a time. How far would he run in bringing back all the potatoes? [NCERT EXEMPLAR]


Answer:

Given: total potatoes are 20. First potato is 24 meters away from starting point equal distance between two adjacent potatoes is 4 meters


To find: the total distance run by contestant


As contestant is coming back to starting point after bringing every potato


Distance covered by him to bring 1st potato and return to the initial position is 24m + 24m = 48m


Now, distance between adjacent trees is 4m


Distance covered by him to bring 2nd potato and return to the initial position is 28m + 28m = 56m


Distance covered by him to bring 3rd potato return to the initial postion is 32m + 32m = 64m


Hence distance covered by him to bring the potatoes are in A.P


A.P. is 48, 56, 64………………………………………upto 20 terms


Here first term, a = 48, common difference, d = 56 – 48 = 8


And n = 20


We need to find S20 which will be the total distance covered by contestant to bring 20 potatoes


Formula used:



where a is first term, d is common difference and n is number of terms in an A.P.


Therefore,






⇒ S20 = 2480


Hence, total distance covered by contestant to bring potatoes is 2480 m



Question 15.

A man accepts a position with an initial salary of ₹ 5200 per month. It is understood that he will receive an automatic increase of ₹ 320 in the very next month and each month thereafter.

(i) Find hi salary for the tenth month.

(ii) What is his total earnings during the first year?


Answer:

Given: Initial salary is Rs. 5200 per month and will increase Rs. 320 every month i.e. a = 5200 and d = 320


(i) Find hi salary for the tenth month.


Answer:


To find: Salary for the tenth month i.e. a10


Formula used:


an = a + (n – 1)d


⇒ a10 = 5200 + (10 – 1)320


⇒ a10 = 5200 + (9)320


⇒ a10 = 5200 + 2880


⇒ a10 = 8080


Hence, he will get Rs. 8080 in tenth salary


(ii) What is his total earnings during the first year?


Answer:


To find: His total earnings during the first year i.e. 12 months, S12


Formula used:



where a is first term, d is common difference and n is number of terms in an A.P.


Therefore,






⇒ S12 = 83520


Hence, his total earnings during an year is Rs. 83520



Question 16.

A man saved ₹ 66000 in 20 years. In each succeeding year after the first year he saved ₹ 200 more than what he saved in the previous year. How much did he saved in the first year?


Answer:

Given: A man saved ₹ 66000 in 20 years


To find: His saving in first year i.e. a


According to question:


n = 20, S20 = 66000 and d = 200


Formula used:



where a is first term, d is common difference and n is number of terms in an A.P.


Therefore,






⇒ 2a = 6600 – 3800


⇒ 2a = 2800



⇒ a = 1400


Hence, his saving in first year is Rs. 1400



Question 17.

In a cricket team tournment 16 teams particpated a sum of 8000 is to to awarded among themselves a price money. It the last placed team is awarded 275 in price money and the award increased by the same amount for successive finishing place , how much amount will the first place team receive?


Answer:

Given: Total teams in tournament are 16


Total prize money is Rs 8000


The last placed team is awarded Rs. 275


To find: Amount received by first place team


As the award money is increased by the same amount for successive finishing place, it forms an A.P.


Let a is amount receievd by first place team and d is successive increase in award


A.P. will be a, a + d, a + 2d,……………………


According to question:


n = 16, S16 = 8000 and a16 = 275


Formula used:


an = a + (n – 1)d



where a is first term, d is common difference and n is number of terms in an A.P.


Therefore,


a16 = a + (16 – 1)d


⇒ 275 = a + 15d


⇒ a = 275 – 15d …………(1)






⇒ 1000 = 2a + 15d …………(2)


Now put the value of a from (1) in (2):


⇒ 1000 = 2(275 – 15d) + 15d


⇒ 1000 = 550 – 30d + 15d


⇒ 1000 = 550 – 15d


⇒ 15d = 550 – 1000


⇒ 15d = –450



⇒ d = -30


Put d = -30 in (1):


a = 275 – 15(-30)


⇒ a = 275 + 450


⇒ a = 725


Hence, amount received by first placed team is Rs. 725




Very Short Answer
Question 1.

Write the common difference of an A.P. whose nth term is xn + y.


Answer:

Here, an=xn+ y


∴ a1=x + y


And a2=2x+y


So, common difference of an A.P. is given by


d=a2–a1


=(2x+y)-(x+y)


=x



Question 2.

Write the common difference of an A.P. the sum of whose first n terms is .


Answer:

Here,


So,


And


=2P+2Q


So,




Now, common difference of A.P. is given by


d=t2-t1






Question 3.

If the sum of n terms of an AP is , then write its nth term.


Answer:

Here, Sn=2n2+3n


So, S1=2+3=5=t1


And S2=2×22+3×2


=8+6=14


So, t2=S2-S1


=14-5


=9


Now, common difference of A.P. is given by


d=t2-t1


=9-5


=4


So, nth term of A.P. is


an=t1+(n-1)d


=5+(n-1)4


=5+4n-4


=4n+1



Question 4.

If log 2, and are in A.P., write the value of x.


Answer:

Here, log 2, log(2x-1) and log(2x+3) are in A.P.


So, log(2x-1)-log2=log(2x+3)-log(2x-1)





Let 2x=y


Then above equation is written as


(y-1)2=2(y+3)


∴y2-2y+1=2y+6


∴y2-4y-5=0


∴(y-5)(y+1)=0


∴ y=5 or y=-1


∴ 2x=5 or 2x=-1


∴ x=log25 or 2x=-1 is not possible


∴ x=log25



Question 5.

If the sum of n terms of two arithmetic progressions are in the ratio 2n + 5 : 3n + 4, then write the ratio of their mth terms.


Answer:

Let nth term of first A.P. be denoted by an


And nth term of second A.P. be denoted by an



Now, multiplying both sides by 2 we get,








The ratio of mth term is 4m+3:6m+1



Question 6.

Write the sum of first n odd natural numbers.


Answer:

Here, A.P is 1,3,5…


So, a=1 and d=3-1=2


Now, sum of n term is given by,








Question 7.

Write the sum of first n even natural numbers.


Answer:

Here, A.P is 2,4,6…


So, a=2 and d=4-2=2


Now, sum of n term is given by,






=n(n+1)



Question 8.

Write the value of n for which nth of the A.P.s 3, 10, 17, ... and 63, 65, 67, ... are equal.


Answer:

Let nth term of A.P. 3,10,17,… be an


And nth term of A.P. 63,65,67,…be an


So,a1=3 and a2=10


∴ d=a2-a1=10-3=7


And, a1’=63 and a2’=65


∴ d’=a2’-a1’=65-63=2


Now, an=a1+(n-1)d


=3+(n-1)7


=3+7n-7


=7n-4


And, an’=a1’+(n-1)d’


=63+(n-1)2


=63+2n-2


=2n+61


Now,it is given that an=an


∴ 7n-4=2n+61


∴ 7n-2n=61+4


∴ 5n=65


∴ n=13



Question 9.

If , then find the value of n.


Answer:

Here,


i.e.



(∵a=3;d=2;a’=5;d’=3)



∴n(2+n) =7×5×37


∴n2+2n-1295=0


∴(n-35) (n+37) =0


∴ n=35 or n=-37 which is not possible as n is non negative.


So,n=35



Question 10.

If mth term of an A.P. is n and nth term is m, then write its pth term.


Answer:

Here, am=n and an=m


∴ a+(m-1)d=n and a+(n-1)d=m_______(1)


Subtracting above two equation we get


a +(m-1)d-a-(n-1)d=n-m


∴ md-d-nd+d=n-m


∴ d(m-n)=n-m


∴ d=-1


Substituting d=-1 in a+(m-1)d=n we get


∴ a+(m-1)(-1)=n


∴ a-m+1=n


∴ a=m+n-1


Now, pth term is given by ap=a+(p-1)d


=m+n-1+(p-1)(-1)


=m+n-1-p+1


=m+n-p



Question 11.

If the sum of n terms of two A.P.’s are in the ratio (3n + 2) : (2n + 3), find the ratio of their 12th terms.


Answer:

Here,



_____(1)


Now,we want to find ratio of 12th term,


i.e.


___(2)


From (1) and (2)we get n-1=22


i.e.n=23






Mcq
Question 1.

Mark the correct alternative in the following:

If 7th and 13th terms of an A.P. be 34 and 64 respectively, then its 18th term is

A. 87

B. 88

C. 89

D. 90


Answer:

Here,a7=34 and a13=64


i.e. a+6d=34____(i) and a+12d=64_____(ii)


Subtracting (i) from (ii) we get,


(12-6)d=64-34


∴6d=30


∴ d=5


Substituting d=5 in (i) we get,


a+6(5)=34


∴a+30=34


∴ a=4


Now,18th term is given by a18=a+17d


=4+17×5


=89


Question 2.

Mark the correct alternative in the following:

If the sum of p terms of an A.P. is q and the sum of q terms is p, then the sum or p + q terms will be

A. 0

B. p − q

C. p + q

D. − (p + q)


Answer:

Here, Sp=q and Sq=p


i.e. and


i.e. ____(i)


and ____(ii)


Now,multiplying (i) with q and (ii) with p and then subtracting we get


pq(p-1)d-pq(q-1)d=2q2-2p2


∴pqd(p-q)=2(q2-p2)


∴ pqd(p-q)=2(p-q)(p+q)



Substituting the value of d in (i) we get,



∴ 2ap2q-2p(p-1)(p+q)=2pq2


∴ apq-(p-1)(p+q)=q2


∴ apq=q2+(p-1)(p+q)


∴ apq=q2+p2+pq-p-q



Now,




=-(p + q)


Question 3.

Mark the correct alternative in the following:

If the sum of n terms of an A.P. be 3 and its common difference is 6, then its first term is

A. 2

B. 3

C. 1

D. 4


Answer:

Here, Sn=3n2-n


Then, S1=3(12)-1


=2


So,first term is a1=S1=2


Question 4.

Mark the correct alternative in the following:

Sum of all two digit numbers which when divided by 4 yield unity as remainder is

A. 1200

B. 1210

C. 1250

D. none of these


Answer:

Two digits numbers which when divided by 4 yield unity as remainder is A.P. as 13, 17, 21,…,97


In this A.P., first term is a=13


Last term is l=97


And d=17-13=4


Now, l=an=a+(n-1)d


∴ 97=13+(n-1)4


∴ 84=(n-1)4


∴ 21=n-1


∴ n=22


Now,



=11×110


=1210


Question 5.

Mark the correct alternative in the following:

In A.M.’s are introduced between 3 and 17 such that the ratio of the last mean to the first mean is 3 : 1, then the value of n is

A. 6

B. 8

C. 4

D. none of these


Answer:

Let a=4 ;b=17


Let a1,a2,a3,…an be the n A.M’s between 3 and 17


Then, common difference is given by





Now,





And





Now, given that








Question 6.

Mark the correct alternative in the following:

If denotes the sum of first n terms of an A.P. such that , then

A.

B.

C.

D.


Answer:

Here,








Now,





Question 7.

Mark the correct alternative in the following:

The first and last terms of an A.P. are 1 and 11. If the sum of its terms is 36, then the number of terms will be

A. 5

B. 6

C. 7

D. 8


Answer:

Here,a=1 and l=11 and Sn=36


Now,






Question 8.

Mark the correct alternative in the following:

If the sum of n terms of an A.P., is 3 which of its terms is 164?

A. 26th

B. 27th

C. 28th

D. none of these


Answer:

Here, Sn=3n2+5n


∴ S1=3+5=8


∴ a1=S1=8


And S2=3×22+5×2=22


Now,a2=S2-S1=22-8=14


∴ d=a2-a1=14-8=6


Now, an=a1+(n-1)d


∴ 164=8+(n-1)6


∴ 156=(n-1)6


∴ 26=n-1


∴ n=27


Question 9.

Mark the correct alternative in the following:

If the sum of n terms of an A.P. is 2, then its nth term is

A. 4n − 3

B. 3n − 4

C. 4n + 3

D. 3n + 4


Answer:

Here, Sn=2n2+5n


∴ S1=2+5=7


∴ a1=S1=7


And S2=2×22+5×2=18


Now,a2=S2-S1=18-7=11


∴ d=a2-a1=11-7=4


Now, an=a1+(n-1)d


=7+(n-1)4


=7+4n-4


=4n+3


Question 10.

Mark the correct alternative in the following:

If are in A.P. with common difference d, then the sum of the series sin d is

A.

B.

C.

D.


Answer:

Here,a1, a2,…,an are in A.P.


Then d=a2-a1=a3-a2=…=an-an-1








Question 11.

Mark the correct alternative in the following:

In the arithmetic progression whose common difference is non-zero, the sum of first 3 n terms is equal to the sum of next n terms. Then the ratio of the sum of the first 2 n terms to the next 2 n terms is

A. 1/5

B. 2/3

C. 3/4

D. none of these


Answer:

Here, sum of first 3n terms=sum of next n terms


i.e. S3n=S4n-S3n


i.e. 2S3n=S4n






----------(1)


Sum of 2n terms is S2nand sum of next 2n terms is given by S4n-S2n


Now,





(∵ using (1))




So, ratio of the sum of first 2n terms to the next 2n terms is 1:5


Question 12.

Mark the correct alternative in the following:

If are in A.P. with common difference d, then the sum of the series , is

A.

B.

C.

D.


Answer:

Here,a1, a2,…,an are in A.P.


Then d=a2-a1=a3-a2=…=an-an-1








Question 13.

Mark the correct alternative in the following:

If four numbers in A.P. are such that their sum is 50 and the greatest number is 4 times the least, then the numbers are

A. 5, 10, 15, 20

B. 4, 10, 16, 22

C. 3, 7, 11, 15

D. none of these


Answer:

Let 4 numbers of A.P. be the a, a+d, a+2d, a+3d.


Here, given that their sum is 50.


i.e. a+a+d+a+2d+a+3d=50


∴ 4a+6d=50


∴ 2a+3d=25_____(1)


Also, given that the greatest number is 4 times the least.


i.e.4a=a+3d


∴ 3a=3d


∴ a=d


Substituting a=d in (1) we get,


5a=25


∴ a=5


And, d=a=5


So, numbers are a=5;a+d=5+5;a+2d=5+2(5);a+3d=5+3(5)


i.e. 5,10,15,20


Question 14.

Mark the correct alternative in the following:

In n arithmetic means are inserted between 1 and 31 such that the ratio of the first mean and nth mean is 3 : 29, then the value of n is

A. 10

B. 12

C. 13

D. 14


Answer:

Let a=1;b=31


Let a1,a2,a3,…an be the n A.M’s between 1 and 31


Then, common difference is given by





Now,





And





Now, given that








Question 15.

Mark the correct alternative in the following:

Let denote the sum of n terms of an A.P. whose first term is a. If the common difference d is given by , then k =

A. 1

B. 2

C. 3

D. none of these


Answer:

We know Sn=Sn-2+an-1+an


=Sn-2+a+(n-2)d+a+(n-1)d


=Sn-2+2a+2nd-3d


Also, Sn-1=Sn-2+an-1


=Sn-2+a+(n-2)d


=Sn-2+a+nd-2d


Now, d=Sn-kSn-1+Sn-2


= Sn-2+2a+2nd-3d-k(Sn-2+a+nd-2d)+Sn-2


=(2-k)[Sn-2+a+nd]+d(2k-3)


Comparing coefficient of both the side we get,


2-k=0 and 2k-3=1


∴ k=2


Question 16.

Mark the correct alternative in the following:

The first and last term of an A.P. are a and l respectively. If S is the sum of all terms of the A.P. and the common difference is given by , then k =

A. S

B. 2S

C. 3S

D. none of these


Answer:

Here, an=l


∴ a+(n-1)d=l


∴ (n-1)d=l-a



Now, Sn=S










______(1)


Now, given that ______(2)


Comparing (1) and (2) we get


k=2S


Question 17.

Mark the correct alternative in the following:

If the sum of first n even natural numbers is equal to k times the sum of first n odd natural numbers, then k =

A.

B.

C.

D.


Answer:

Let first A.P is 2,4,6…


So, a=2 and d=4-2=2


Let S1 denotes the sum of n even numbers


Now, sum of n term is given by,







Let second A.P is 1,3,5…


So, a=1 and d=3-1=2


Let S2 denotes the sum of n odd numbers


Now, sum of n term is given by,







Now, given that S1=kS2


∴n(n+1)=kn2



Question 18.

Mark the correct alternative in the following:

If the first, second and last term of an A.P. are a, b and 2a respectively, then its sum is

A.

B.

C.

D. none of these


Answer:

Here,a1=a;a2=b and l=2a


Then d=a2-a1=b-a


So,l=an=a1+(n-1)d


∴ 2a=a+(n-1)(b-a)


∴ a=n(b-a)-(b-a)


∴ a=n(b-a)-b+a


∴ n(b-a)=b



Now,


)




Question 19.

Mark the correct alternative in the following:

If, is the sum of an arithmetic progression of ‘n’ odd number of terms and the sum of the terms of the series in odd places, then

A.

B.

C.

D.


Answer:

Here, First A.P is 1,3,5…


So, a=1 and d=3-1=2


Now, sum of n term is given by,







Now, second A.P. is 1,5,9,…


In this A.P. a=1, d=5-1=4


Let us assume that total no. of term in first A.P. is even then total no. of term in second A.P. is


Now,



)




Then



Let us assume that total no. of term in first A.P. is odd then total no. of term in second A.P. is


Now,



)




Then



Question 20.

Mark the correct alternative in the following:

If in an A.P., and , where denotes the sum of r terms of the A.P., then is equal to

A.

B. mn p

C.

D.


Answer:

Here, Sn=n2p


Now, substituting n=p we get


Sp=p2p=p3


Question 21.

Mark the correct alternative in the following:

If in an A.P., the pth term is q and term is zero, then the term is

A. − p

B. p

C. p + q

D. p − q


Answer:

Here, ap=q and a(p+q)=0


i.e. a+(p-1)d=q______(1)


and a+(p+q-1)d=0_____(2)


Subtracting (1) from (2) we get,


qd=-q


∴ d=-1


substituting this in (2) we get,


a+(p+q-1)(-1)=0


∴ a=p+q-1


So, aq=a+(q-1)d


=p+q-1+(q-1)(-1)


Question 22.

Mark the correct alternative in the following:

The 10th common term between the A.P.s 3, 7, 11, 15, ... and 1, 6, 11, 16, ... is

A. 191

B. 193

C. 211

D. none of these


Answer:

Here, common difference of first A.P. is d=7-3=4


And common difference of second A.P. is d’=6-1=5


Now L.C.M of d and d’=(4,5)=20


Also first common term in both A.P.s is 11.


We have to find 10th common term i.e. n=10


Consider an A.P. in which a=11,d=20 and n=10


Then, a10=a+(10-1)d


=11+9×20


=11+180


=191


Question 23.

Mark the correct alternative in the following:

If in an A.P. and , where denotes the sum r terms of the A.P., then equals

A.

B. mnq

C.

D.


Answer:

Here, Sn=n2q


Now, substituting n=q we get


Sq=q2p=q3


Question 24.

Mark the correct alternative in the following:

Let denote the sum of first n terms of an A.P. If , then is equal to

A. 4

B. 6

C. 8

D. 10


Answer:

Here,S2n=3Sn







Now,







∴ S3n:Sn=6:1